UWorld Review- OBGYN

Réussis tes devoirs et examens dès maintenant avec Quizwiz!

D. imperforate hymen Imperforate hymen is an anatomic cause of primary amenorrhea. Pubertal patients typically present with cyclic lower abdominal pain, amenorrhea & hematocolpos. Pelvic exam typically reveals a smooth, blue, bulging vaginal mass that sweeps with increased intraabdominal pressure.

13-year-old girl is brought to the office due to lower abdominal pain for the past 4 days. The patient has rectal pain with bowel movements, but no melon or frank blood. She has had this pain several times over the past year & it usually resolves after 2 or 3 days. The patient has not had a menstrual period & has never been sexually active. Temp is 36.7C (98F) and BP: 110/70; BMI: 24kg/m2. PE shows a non distended abdomen without rebound or guarding. There is lower abdominal tenderness to deep palpation in the midline. The external genitalia are Tanner stage 3. On pelvic exam, a smooth, firm mass is protruding between the labia major. Which of the following is the most likely diagnosis? A. cervical leiomyoma B. complete mullein agenesis C. foreign body D. imperforate hymen E. pelvic organ prolapse

A. Age Women > 35 y/o who smoke have an increased risk of stroke with combined hormonal contraceptives. These women should be prescribed progestin-only or non-hormonal forms of contraception.

39-year-old woman undergoing ABX therapy for PID requests "the pill" for contraception. Medical Hx is significant for uterine fibroids & an ectopic pregnancy treated with Methotrexate. The patient smokes a pack of cigarettes daily but does not use alcohol or illicit drugs. Which of the following is a contraindication for prescribing combo OCPs in this patient? A. Age B. Concurrent ABX use C. Hx of ectopic pregnancy D. PID E. Uterine fibroids

E. Screening is appropriate in all women of childbearing age regardless of relationship status Intimate partner violence (IPV), which includes both physical/sexual & psychological abuse, is more common in women. Due to high rates of IPV & underreporting, all women of childbearing age should undergo routine IPV screening (regardless of risk factors).

In which of the following scenarios would intimate partner violence screenings be most appropriate? A. Annual exam for a 40-y/o woman who has been married for 15 years & has 2 children B. Contraceptive counseling visit for a 17-y/o girl with 2 new male sexual partners over the past 6 months C. In vitro fertilization consult for a 33-y/o woman in a monogamous same-sex relationship for the past 5 years D. Initial prenatal visit for 28-y/o primigravida at 12 weeks gestation with a boyfriend of 1 year E. Screening is appropriate in all women of childbearing age regardless of relationship status

A. Dietary modification this is first-line treatment for mild nausea of pregnancy. Pregnant women should avoid offending odors & eat small, frequent meals consisting of bland carbs or protein-based snacks.

20 year old woman presents due to nausea. She has had a decreased appetite & felt "queasy" for the past 2 weeks. She has had no vomiting, abdominal pain or diarrhea. The patient's LMP was 9 weeks ago. VS & physical exam are normal. A pregnancy test is positive & UA is negative for keytones. US confirms a viable intrauterine gestation. Which of the following is the best next step in management of this patient's nausea? A. Dietary modification B. Doxylamine C. Hydroxyzine D. Ondansetron E. Prochlorperazine

C. Naproxen therapy Primary dysmenorrhea, lower abdominal cramping associated with menstrual periods, is common in adolescents & is due to excessive prostaglandins production. The 1st-line management of primary dysmenorrhea in non-sexually active patients is NSAIDs (Naproxen).

15-year-old girl is brought to the office for eval of lower abdominal pain associated with her menses that has occurred over the past few months. She describes the pain as "dull" & "crampy" and radiating to her back & lower thighs. The pain is associated with nausea & bloating. She has tried using a heating pad with minimal relief of symptoms & has missed several days of school due to pain. The patient has no pain between periods. Her menses started at age 14, have occurred every month & last 5 days. The girl has never had sexual intercourse. BMI is 25kg/m2. Pelvic exam reveals a small nontender uterus without masses. A small amount of clear d/c is seen throughout the vaginal vault. The cervix has no lesions or areas of friability. Which of the following is the best next step in management of this patient's symptoms? A. CA-125 level B. Combined oral contraceptives C. Naproxen therapy D. Pelvic US E. Wet mount microscopy

B. Intracellular diplococci Microscopy is not preferred in the diagnosis of acute cervicitis due to its low sensitivity in detecting Neisseria gonorrhoeae (a gram-negative intracellular diplococci) & inability to detect Chlamydia trachoma's (an obligate intracellular bacteria). Nucleic acid amplification testing is the gold standard for diagnosis.

19-year-old nulligravid woman comes to the office due to abnormal vaginal d/c for the last 2 weeks. She had an episode of postcoital vaginal bleeding a few days ago, followed by return of the abnormal yellow d/c. PE shows mucopurulent cervical d/c. The cervix is friable & bleeds easily on cotton tip manipulation. Urine pregnancy test is negative. Nucleic acid amplification testing is not available. A sample of the d/c is obtained for microscopic exam. Which of the following is the most likely finding in this patient? A. Clue cells B. Intracellular diplococci C. Pseudohyphae D. Spirocetes E. Trichomonads

B. HPV vaccination alone HPV vaccination is recommended to prevent HPV-related disease; it is typically administered to those age 11-26 but can be given from 9-45. Cervical cancer screening with Pap testing begins at age 21 in immunocompetent patients regardless of age of onset of sexual activity.

20-year-old woman comes to the office for a refill of oral contraceptives. She is in a monogamous relationship that began 6 months ago & uses condoms as back-up contraception. The patient became sexually active at age 14 & has had 5 lifetime partners. Her LMP was 2 weeks ago, and she has regular menses. The patient has not received the human papilloma virus (HPV) vaccination series & has never had a Pap test. The patient has no chronic medical conditions. PE is normal. Which of the following is recommended for this patient? A. HPV testing and, if negative, HPV vaccination B. HPV vaccination alone C. Pap testing alone D. Pap with HPV testing E. Reassurance & follow-up in 1 year

D. Proceed to cesarean delivery Active phase arrest is no cervical change in > 4 hours with adequate contractions (>200 Montevideo units averaged over 10 mins), or no change in > 6 hours with inadequate contractions. Labor arrest is managed by cesarean delivery.

23-year-old woman, gravida 1 para 0, at 41 weeks gestation is admitted to labor & delivery for contractions over the past 3 hours. Temp is 36.7C (98F). Her cervical exam shows 8cm dilation, unchanged over the last 4 hours. The patient's labor curve shows she dilated from 5 cm to 7 cm in the first 2 hours, crossing from the latent phase into the active phase. Once in the active phase, she dilated from 7 cm to 8 cm in 2 hours (appropriate rate of dilation of 1 cm/2hrs). FHR tracing shows a baseline of 140/min, moderate variability & multiple accelerations. Intrauterine pressure catheter shows an average of 240 Montevideo units every 10 mins for the last 4 hours, consistent with adequate contractions. The patient is diagnosed in active phase arrest of labor. Which of the following is the best next step in management of this patient? A. Administer prostaglandin B. Continue current management C. Operative vaginal delivery D. Proceed to cesarean delivery E. Rotate fetus to breech presentation

D. Transvaginal US This patient has PAINLESS vaginal bleeding > 20 weeks gestation, a presentation consistent with placenta previa. Placenta previa, the extension of the placenta over the internal cervical os, is associated with increased risk of obstetrical hemorrhage & preterm labor. B/c the bleeding from placenta previa is maternal in origin, the fetal heart rate tracing is typically normal. Requires a transvaginal US to determine. Digital cervical exam is contraindicated.

23-year-old woman, gravida 3 para 2, at 35 weeks gestation comes to the ED due to vaginal bleeding that started after intercourse, 2 hours prior to arrival, & has soaked through 3 perineal pads. The patient reports normal fetal movement & mild intermittent cramping but no leakage of amniotic fluid. Her only prenatal visit was at 7 weeks gestation. The patient has a history of IV drug use. BP is 130/70, HR is 98/min. The fetal heart rate is normal. The tachometer shows contractions every 5 mins. On exam, the patient is in no distress. The abdomen is gravid, soft & nontender. There is frank blood on the patient's perineal pad. Which of the following is the best next step in management of this patient? A. Digital cervical exam B. Fetal fibronectin testing C. Induction of labor D. Transvaginal US E. Urine toxicology screen

B. ABX due to the # of bacterial colonies in urine Pregnant women are universally screened for bacteriuria at 12-16 weeks gestation. Pregnant patients with asymptomatic bacteriuria (>100,000 colony-forming units/mL in a urine culture with no associated symptoms) require treatment to prevent acute pyelonephritis.

24-year-old woman, gravida 2 para 1, at 12 weeks gestation comes to the office for a prenatal visit. The patient has had no abdominal pain, vaginal bleeding, dysuria or hematuria. Vitals are normal. Fetal heart rate is 150/min by Doppler US. Results from a clean catch urine sample- Specific gravity: 1.025 (normal: Protein: trace (normal: Glucose: negative Ketones: negative Leukocyte esterase: negative Nitrites: negative Bacteria: many Urine culture reveals > 100,000 colony-forming units/mL of E. coli. Which of the following is the most appropriate management strategy for this patient? A. ABX due to the associated proteinuria B. ABX due to the # of bacteria colonies in urine C. No ABX due to the current gestational age D. No ABX given the lack of symptoms E. No ABX given the negative leukocyte esterase & nitrites

E. Place the patient in the left lateral position Management of a late fetal heart rate deceleration includes intrauterine resuscitation measures (maternal repositioning) & correction of the underlying etiology. If the fetal heart rate tracing persists or worsens, emergency delivery is recommended.

24-year-old woman, gravida 2 para 1, at 39 weeks gestation is admitted in active labor. Her pregnancy has been uncomplicated. The patient received an epidural on admission, and the pain has improved. Vitals are normal. The cervix is 6 cm dilated & 100% effaced & the fetal head is at 0 station. The fetal heart rate tracing shows a baseline of 140/min, minimal variability & multiple decelerations with a nadir that occurs after the peak of the contraction. Which of the following is the best step in management of this patient? A. Continue current management B. Decrease the epidural infusion rate C. Perform an emergency cesarean delivery D. Place an intrauterine pressure catheter E. Place the patient in the left lateral position

C. Flex maternal thighs back against the abdomen Shoulder dystocia is an obstetric emergency in which the fetal shoulders are impacted in the maternal pelvis, requiring additional maneuvers to dislodge to deliver the baby. Flexing the mother's thighs toward the abdomen (McRoberts maneuver) while she is supine is the first step in management.

26-year-old woman, gravida 2 para 1, at 42 weeks gestation undergoes induction of labor. The 1st stage of labor lasts 22 hours. After 30 mins of the 2nd stage, the fetal head emerges but immediately retracts into the perineum. Gentle downward traction is applied to the fetal head during the next contraction while the patient pushes; however, the anterior shoulder does not deliver. Which of the following is the most appropriate next step in management? A. Apply increased downward traction on the fetal head B. Apply vacuum for operative delivery C. Flex maternal thighs back against the abdomen D. Locate & deliver the posterior fetal arm E. Perform emergency cesarean delivery

E. "There is no risk of hemolytic disease to your baby from Rh incompatibility." Hemolytic disease of the newborn due to Rh(D) incompatibility is possible only in a Rh(D)-negative mother & Rh(D)- positive father.

27-year-old woman, gravida 3 para 1 aborta 1, at 20 weeks gestation comes to the office as she is concerned that she might be at risk for hemolytic disease. She just learned that her sister-in-law's 2nd baby has hydros fetalis due to Rh(D) incompatibility & is concerned that this could happen to her 2nd baby as well. The patient's pregnancy has been uncomplicated to date. The patient is Rh(D) positive & her husband is Rh(D) negative. Their 3-year-old son is healthy, and he is Rh(D) positive. A year prior, the patient had a missed abortion requiring suction curettage. She has no chronic medical problems & no other surgeries. Which of the following is the most accurate statement to the patient regarding hemolytic disease due to Rh(D) incompatibility? A. "The risk of hemolytic disease disease for your baby can be analyzed using the results of serial US." B. "The risk of hemolytic disease for your depends on whether you previously received Rho(D) immune globulin." C. "The risk of hemolytic disease occurring in this pregnancy is dependent on the baby's Rh blood group." D. "There is significant risk of hemolytic disease to your baby due to Rh incompatibility." E. "There is no risk of hemolytic disease to your baby from Rh incompatibility."

A. Endocervical curettage Patients with high-grade squamous intraepithelial lesions on Pap testing are at high risk for cervical intraepithelial neoplasia & cervical cancer. B/c these lesions typically arise from the transformation zone, patients with unsatisfactory colposcopy results (entire squamocolumnar junction cannot be visualized) require endocervical sampling (endocervical curettage).

31-year-old woman comes to the office for follow-up due to an abnormal Pap test. 2 weeks ago, Pap testing revealed a high-grade squamous epithelial lesion. The patient has no chronic medical conditions & has had no prior surgeries. She has regular monthly menses, & her LMP was 1 week ago. BMI is 33g/m2. Colposcopy reveals a nulliparous cervix with no raised or ulcerated lesions. The entire squamocolumnar junction cannot be visualized. Which of the following is the best next step in management of this patient? A. Endocervical curettage B. Endometrial biopsy C. HPV testing in 5 years D. Repeat Pap testing in 1 year E. Transvaginal pelvic US

A. Cefoxitin + Doxycycline PID presents with fever, abdominal pain, cervical motion tenderness & mucopurulent cervical d/c. PID is a polymicrobial infection due to Chlamydia trachoma's, Neisseria gonorrhoeae & vaginal flora. Treatment is with broad-spectrum ABX: Cefoxitin (a 2nd generation cephalosporin) + Doxycycline

31-year-old woman is brought to the ED due to fever, chills, abdominal pain & urinary frequency for the past 5 days. She vomited twice today, and her symptoms have become progressively worse. The patient reports increased malodorous vaginal d/c but no abnormal vaginal bleeding. Her LMP was 1 week. The patient is sexually active & uses OCPs. She has no known drug allergies. Temp is 38.9C (102F), BP is 100/60, HR is 110/min & RR are 18/min. The patient is pale & diaphoretic. The abdomen is soft, diffusely tender & non-distended; bowel sounds are increased. On pelvic exam, there is light-yellow d/c at the external cervical os & tenderness on lateral movement of the cervix. Which of the following is the most appropriate management of this patient? A. Cefoxitin + Doxycycline B. Ceftriaxone + Metronidazole C. Ciprofloxacin + Metronidazole D. No ABX until lab results are back E. Piperacillin + Tazobactam

A. Aspirin Patients with a history of renal disease, diabetes mellitus, HTN, autoimmune disease, or prior preeclampsia (particularly preterm preeclampsia with severe features) are at high risk for preeclampsia recurrence. In high-risk patients, aspirin is used for preeclampsia prevention & is started at 12 weeks gestation.

32-year-old, gravida 2 para 1, at 12 weeks gestation comes to the office for an initial prenatal visit. The patient's previous pregnancy was complicated by an induction of labor at 36 weeks gestation for preeclampsia with severe features. The patient has no chronic medical conditions. BP is 114/78, HR is 88/min. BMI is 23kg/m2. PE is unremarkable. Pelvic US reveals a singleton intrauterine pregnancy with a normal fetal heart rate. Which of the following is recommended to reduce the risk of preeclampsia in this pregnancy? A. Aspirin B. Fish oil C. Folic acid D. Labetalol E. Lisinopril

C. Placenta previa Placenta previa occurs when the placenta covers the cervix & presents with painless vaginal bleeding after 20 weeks gestation. Blood loss is primarily maternal; therefore, initial fetal monitoring is typically reassuring.

34-year-old woman, gravida 4 para 2 aborta 1, comes to the ED at 24 weeks gestation with vaginal bleeding. The patient woke up in a pool of bright red blood & continues to soak through a pad every hour. She has not had routine prenatal care during this pregnancy. The patient has had 2 cesarean deliveries at term. BP is 124/78, HR is 92/min. Fetal monitoring shows a baseline of 140/min, accelerations, no decelerations & moderate variability. Irregular contractions are seen on tocodynamoqmetry. The uterus is nontender. On speculum exam, there are large clots in the vaginal vault, a closed cervix & active bleeding from the os. Which of the following is the most likely diagnosis in this patient? A. Cervical insufficiency B. Inevitable abortion C. Placenta previa D. Placental abruption E. Preterm labor

D. Oxytocin Uterine atony is the m/c cause of postpartum hemorrhage & is typically due to uterine fatigue from prolonged labor induction. Initial treatment includes Oxytocin & uterine massage.

36-year-old woman, gravida 1 para 1 underwent a spontaneous vaginal delivery of a 3.6kg (7lb 15oz) infant after an uncomplicated induction of labor for chronic HTN. The placenta delivers intact 5 mins later with controlled cord traction & uterine massage. The vagina is inspected & a second-degree perineal laceration is repaired in standard fashion. After completion of the repair, the uterus feels boggy & the patient begins to pass large clots. Which of the following is the best next step in management of this patient? A. Intrauterine ballon tamponade B. Methylergonovine C. Misoprostol D. Oxytocin E. Sharp curettage

A. Colposcopy & endometrial biopsy Atypical glandular cells (AGC) on a Pap test may be due to either cervical or endometrial adenocarcinoma. Eval of AGC requires colposcopy, endocervical curettage & endometrial biopsy.

42-year-old woman comes to the office for a routine health exam. she is sexually active with a male partner & underwent a tubal ligation for contraception. The patient has had no abnormal bleeding or recent changes in weight. menses are regular & last 3-5 days; her LMP was 3 weeks ago. Pap tests have been normal to date. Pelvic exam shows a normal cervix without any visible lesions. The Pap test shows atypical gladular cells. Which of the following is the next best step in management of this patient? A. Colposcopy B. Hysterectomy C. Loop electrosurgical excision procedure (LEEP) D. Return to routine Pap screening E. Serum CA-125

D. Reassurance only Manopausal transition is characterized by menstrual irregularity & hypoestrogenic symptoms (vasomotor symptoms, sleep disturbances). Women age > 45 with these symptoms require no additional testing & should be provided reassurance.

48-year-old woman comes to the office due to irregular menstrual cycles. Over the last year, the patient's menstrual cycles have increased in length. She currently has an average cycle length of 40 days with 5 days of light bleeding. She previously had 28-day cycles with 4 days of bleeding. The patient feels more irritable at work & also has night sweats & difficulty sleeping 2 or 3 nights a week. PE is normal. A pregnancy test is negative. Which of the following is the best next step in management of this patient? A. Coagulation studies B. Pelvic US C. Prolactin level D. Reassurance only E. TSH level

E. Weight loss & dietary changes Stress urinary incontinence is the involuntary leakage of urine during increased intraabdominal pressure due to deficient urethral support. The initial management includes lifestyle modifications (weight loss, smoking cessation, decreased intake of caffeine & alcohol). Surgical management is indicated if lifestyle modifications & conservative management do not improve symptoms.

49-year-old woman, gravida 2 para 2, comes to the office for leakage of urine. The patient reports occasional leakage whenever she sneezes, laughs, or carries groceries. She has no chronic medical conditions & no previous surgeries. Her first delivery was to a 4.5-kg (10-lb) infant & was complicated by a 4th-degree perineal laceration. Menses are regular, every 30 days & consist of 2-3 days of moderate bleeding. She drinks 2 cups of coffee in the morning & 1-2 glasses of wine with dinner each night. BMI is 32kg/m2. Speculum exam shows a well-rugged vagina with no cystocele or rectocele. Leakage of urine & excessive urethral motion are noted when the patient is asked to Valsalva. Which of the following is the most appropriate treatment for this patient's condition? A. Botulinum toxin injections B. Hormone replacement therapy C. Oxybutynin D. Urethral sling surgery E. Weight loss & dietary changes

C. Vaginal biopsy Vaginal squamous cell carcinoma typically presents with vaginal bleeding, malodorous d/c & irregular lesion. Risk factors include: age > 60, chronic tobacco use & persistent HPV infection. Diagnosis is by biopsy of the lesion, which determines the depth of invasion of atypical cells.

63-year-old woman comes to the office for eval of postmenopausal bleeding. The patient initially had post-coital bleeding only, but for the last month, the patient has had daily vaginal spotting. Until 6 months ago, she had not been sexually active for 10 years. The patient has had no routine health care for the past 30 years. She smokes a pack of cigarettes daily. BMI is 30kg/m2. Vagina appears atrophic with minimal rogation & has a 1-cm ulcerated lesion in the upper third of the posterior wall. There is a malodorous watery discharge in the vagina. Wet mount microscopy is negative. Pelvic US shows an anteverted uterus with a 3-mm endometrial stripe. Which of the following is the best step in management of this patient? A. Oral Metronidazole B. Reassurance & follow-up in 3 months C. Vaginal biopsy D. Vaginal estrogen E. Wide local excision

C. Overflow incontinence Overflow incontinence presents as a constant dribbling of urine & is due to bladder distension from incomplete emptying. Typical exam findings include signs of neuropathy & an increased post-void residual urine volume (> 150mL). Treatment involves intermittent self-catheterization & correction of the underlying etiology.

69-year-old woman, gravida 5 para 5, comes to the office for leakage of urine. The patient reports that her urine "leaks all the time" & that she wakes up every hour to go to the bathroom. Since menopause at age 53, the patient has had occasional leakage of urine after sneezing, but her symptoms have worsened over the last few months. She has DM2 & is on an insulin regimen. The patient was recently diagnosed with allergic rhinitis & started on a daily antihistamine. BMI is 25kg/m2. Speculum exam shows a pale vagina but no pelvic organ prolapse. Urine dribbling is noted on exam, but no leakage occurs when the patient is asked to cough. There is decreased sensation to pinprick testing over the perineum. Which of the following is the most likely diagnosis in this patient? A. GU syndrome of menopause B. Intrinsic sphincter deficiency C. Overflow incontinence D. Urethral hyper-mobility E. Urgency incontinence

E. Pessary placement Pelvic organ prolapse can present with a vaginal bulge & urinary, defecatory or sexual dysfunction. Patient who are poor surgical candidates can usually be treated with pessary placement.

86-year-old woman comes to the office due to a vaginal bulge & difficulty urinating. Over the last few months, as the bulge has enlarged, she has experienced increasing pelvic pressure. In addition, she now must manually elevate the bulge to urinate. The patient has poorly-controlled COPD & a 50-pack-year smoking Hx. BP is 150/90, HR is 78/min & pulse ox is 91% on room air. The lungs have decreased breath sounds bilaterally. Pelvic exam reveals anterior vaginal wall prolapse & protrusion of the cervix through vaginal Introitus. The patient is asked to cough & no leakage of urine is observed. Post-void residual urine volume is normal. Which of the following is the best next step in management of this patient? A. Hysterectomy B. Intermittent self-catheterization C. Kegal exercises D. Mid-urethral sling procedure E. Pessary placement

C. hypothalamic-pituitary-ovarian axis immaturity In adolescents who have recently undergone menarche, the immature hypothalamic-pituitary-ovarian axis fails to produce appropriate quantities & ratios of gonadotropin-releasing hormone, therefore LH & FSH, to induce ovulation. As a result, during the first few years post menarche, the majority of menstural cycles are anovulatory & present as painless, irregular, heavy bleeding.

14-year-old girl is brought in due to heavy vaginal bleeding. Since menarche at age 13, menses have been irregular but not painful. Her LMP was 6 weeks ago, & her current menses started 7 days ago. She is soaking through a thick pad every 2 or 3 hours & bled through her clothing overnight. She has no history of recurrent epistaxis or bruising. VS are normal. The abdomen has no masses. On pelvic exam, there is dark red bleeding from the cervical os. Urine pregnancy test is negative. Which of the following is the most likely cause of this patient's symptoms? A. acute cervicitis B. adenomyosis C. hypothalamic-pituitary-ovarian axis immunity D. uterine fibroids E. von Willebrand disease

B. FSH level Primary amenorrhea is the absence of menarche in girls age > 13 with no secondary sexual characteristics. In patients with a uterus, the best next step is an FSH level test, which distinguishes between central (low/normal FSH) & peripheral (high FSH) causes of amenorrhea.

14-year-old girl is brought to the office bc she has not started menstruating. She is otherwise healthy & takes no daily medication. Her mother & older sister underwent menarche at age 13. Height is at the 15th percentile and weight is at the 20th percentile for age. Vitals are normal. PE shows no breast development or axillary hair. The abdomen is soft, nontender & non distended. Pelvic exam reveals normal external female genitalia and no pubic hair. US confirms the presence of a uterus. Which of the following is the best next step in management of this patient? A. Estrogen level B. FSH level C. GnRH stimulation test D. Karyotype analysis E. MRI of pituitary

C. Endometrium PCOS presents with hyperandrogegism (severe acne, hirsutism & androgenic alopecia) & irregular menses. Patients with PCOS are at increased risk for endometrial hyperplasia and cancer due to unregulated endometrial proliferation from unopposed estrogen stimulation.

16 yr old girl comes to the office for eval of acne. Around the time of her first menstrual period at age 10, the acne appeared only on her face, but over the past 6 years, it has spread to her chest and back. topical preparations are not effective, and the patient feels anxious regarding social situations at school. She quit the soccer team and has since gained 9.1kg (20lb). Menses are irregular and her LMP was 3 months ago. She has completed the human papilloma virus vaccination series. PE shows cystic and nodular acne over the face, back and upper chest. There is no goiter or thyroid masses. Terminal hairs are notable on the upper lip and around the nipples. Development is Tanner stage V. The abdomen is obese and nontender. Serum total testosterone and dehydroepiandrosterone sulfate levels are normal. This patient is at greatest risk for developing malignancy of which of the following? A. Adrenal gland B. Cervix C. Endometrium D. Thyroid E. Vulva

B. Decrease in mass size after menses An adolescent with a single rubbery, mobile & well-circumscribed breast mass most likely has a fibroadenoma. Breast tenderness & mass size preceding menses typically decreases after menstrual period has ended.

16-year-old girl comes to the office due to a breast lump that she discovered 2 days ago while applying lotion. Menarche was at age 12 & menstrual periods are regular, occurring approx. every 30 days. Her LMP was about 3 weeks ago. The patient has a history of atopic dermatitis that typically flares during the summer & with exercise. PE shows a rubbery, 3-cm mass in the superior outer quadrant of the left breast. Which of the following is most likely to be expected in this patient? A. Axillary LAD B. decrease in mass size after menses C. Eczematous lesions on areola D. Similar mass in the opposite breast E. Symptom resolution with need aspiration

C. Hypothalamic suppression Functional hypothalamic amenorrhea results from decreased gonadotropin-relasing hormone released from the hypothalamus & is caused by weight loss, strenuous exercise, systemic illness or abnormal eating habits. Loss of cyclic gonadotropin release leads to a decrease in LH & FSH secretion from the pituitary, which in turn causes low circulating estrogen levels.

16-year-old girl comes to the office for a pre-participatory school sports exam. She plays on her high school soccer team & also frequently competes in local beauty pageants. The patient follows strict dietary limitations & exercises strenuously 2 hours a day. Onset of menses was at age 13, but her LMP was 8 months ago. She otherwise feels well. Her height is 165cm (5ft 5in) & her weight is 45.3kg (100lb) with BMI of 16.6 kg/m2. PE shows a pale & thin girl with fine hair around her trunk, but the remainder is otherwise normal. Urine pregnancy test is negative. Which of the following is the most likely mechanism of her amenorrhea? A. Hyperprolcactinemia B. Hyperthyroidism C. Hypothalamic suppression D. Primary ovarian failure E. Primary pituitary dysfunction

C. Location of pain Dysmenorrhea (painful menses) is common in reproductive-aged women & is divided into primary (physiologic) & secondary (pathologically) causes. Clinical features suggestive of a secondary cause (eg endometriosis) include age > 25 at onset, unilateral (nonmidline) pelvic pain, lack of systemic symptoms & abnormal uterine bleeding (AUB).

16-year-old girl is brought to the office by her mother for eval of painful menstrual periods. Since menarche, the patient has had increasingly painful menses that now cause her to miss track practice a few days each month. Menstrual periods occur every 25-27 days with 5 days of moderate bleeding. For the first 4 days of her menses, the patient has cramping pain in the RLQ that radiates to the right flank. The pain improves with Ibuprofen & a heating pad. She has fatigue & nausea that start the day prior to menses & resolve when the pain stops. Her LMP was 3 weeks ago. Pelvic exam is deferred but physical exam is unremarkable. Which of the following findings is most concerning for a secondary cause of dysmenorrhea in this patient? A. Age at onset B. Associated symptoms C. Location of pain D. Pattern of menstrual bleeding E. Response to medication

A. androgen insensitivity syndrome is due to a nonfunctioning androgen receptor, resulting in genotypical male (46, XY) patients appearing as phenotypically female. Patients have primary amenorrhea due to lack of female internal genitalia, male-range testosterone levels, normal beast & female external genitalia development & minimal or no axillary & pubic hair.

16-year-old girl presents due to absent menarche. Her height is at the 70th percentile & weight is at the 40th percentile. VS are normal. On PE, there is Tanner Stage 4 breast development with minimal axillary & pubic hair. The external genitalia are normal, the vagina is 3 cm in length & the cervix is not visible. No uterus, cervix or ovaries are palpated on bimanual exam. Lab results include a negative pregnancy test & a testosterone level of 400ng/dL (normal: 15-75 [Female]; 300-1,000 [Male]). Karyotype analysis is 46, XY. Which of the following is the most likely diagnosis in this patient? A. androgen insensitivity syndrome B. klinefelter syndrome C. mullein agenesis D. transverse vaginal septum E. Turner syndrome

D. Levonorgestrel IUD Copper & Levonorgestrel IUDs are long-acting, reversible contraceptives that have the lowest failure rates in pregnancy prevention. IUDs are a 1st line option in adolescents & safe in nulliparous patients.

17-year-old nulligravid girl is brought to the office by her mother for contraceptive counseling. The patient recently became sexually active & is interested in contraception. Next month, the patient will attend an out-of-state university & wants the most reliable option. She received a swimming scholarship & will have a busy training schedule in addition to her school work. Menarche was at age 13 & she has regular menses typically lasting 3-4 days. A urine pregnancy test is negative. Gonorrhea & chlamydia testing is negative. Which of the following is the best contraception option for this patient? A. Combo hormonal patch B. Combo hormonal vaginal ring C. Combo OCPs D. Levonorgestrel IUD E. Medroxyprogesterone injection

E. Metronidazole Diagnosis criteria for bacterial vaginosis include a thin, gray, malodorous vaginal d/c; a vaginal pH > 4.5; an amine odor after the application of KOH (whiff test); and clue cells on wet mount microscopy. Treatment is with Metronidazole or Clindamycin.

18-year-old woman comes to the office for eval of vaginal d.c that is thin, gray & malodorous. The patient has had no vulvovaginal pruritus or pelvic pain. She is sexually active with multiple partners & uses OCPs but does not consistently use a condom. Her general health is good. Vitals are normal. Speculum exam reveals a gray, adherent, malodorous d/c on the walls of the vaginal vault. The cervix appears normal & there is no cervical motion tenderness. Vaginal pH is 5. Microscopic exam shows epithelial cells diffusely coated with bacteria. Which of the following is the best therapy for this patient? A. Azithromycin B. Ceftriaxone C. Doxycycline D. Fluconazole E. Metronidazole

D. OCP with high-dose estrogen the treatment of acute uterine bleeding is combo OCP containing high-dose estrogen, which stabilizes the endometrium & stops the bleeding

18-year-old woman presents to the ED with heavy vaginal bleeding for the past 3 days. The patient has soaked 6 thick sanitary pads today & has had several large blood clots. She has had heavy bleeding for 2 days. The patient denies syncope or palpitations. There is no Hx of recurrent epistaxis, easy bleeding, or bruising. Menarche was at age 14 & her menstrual periods are irregular. The patient has no chronic medical conditions. BP: 120/70; HR: 95/min. Mucous membranes are moist & capillary refill is normal. Pelvic exam reveals bleeding from the cervix after several large clots are evacuated from the vaginal vault. Labs: CBC- Hemoglobin: 11g/dL (normal: 12-16g/dL) Plateletes: 180,000/mm3 (150,000-400,000) Coagulations studies: INR: 0.8 (normal: 1) activated PTT: 35 secs (normal: 11-15 secs) Urine pregnancy test: negative Pelvic US shows small uterus with a thickened endometrium & normal ovaries. Which of the following is the best next step in management of this patient? A. Dilation & curettage B. Endometrial ablation C. IV conjugated estrogen D. OCP with high-dose estrogen E. Progestin-releasing SQ implant

B. Endometriosis Endometriosis, ectopic endometrial tissue implants within the intraabdominal cavity, can cause chronic inflammation & fibrosis. Therefore, patients can develop chronic pelvic pain (cervical motion tenderness), dysmenorrhea, dyspareunia & distorted pelvic anatomy (cervical displacement)

19-year-old woman with DM1 presents for eval of dyspareunia. The patient became sexually active with her boyfriend 4 months ago but has been avoiding intercourse for the past month b/c it has become too painful. The patient is using condoms for contraception. She has regular menses with 3-4 days of heaving bleeding & lower abdominal cramping. VS are normal. On pelvic exam, the vulva & cervix appears non-inflamed. There is a small uterus with a laterally displaced cervix with no cervical motion tenderness. Which of the following is most likely the cause of this patient's dyspareunia? A. Candida albicans vulvovaginitis B. Endometriosis C. Genito-pelvic pain/penetration disorder (vaginismus) D. PID E. Uterine leiomyomata

A. anti-D immune globulin should be administer now & postpartum anti-D immune globulin is administered at 28 weeks gestation to all Rh(D)-negative women with a negative anti-D antibody screen; a repeat dose is given < 72 hours after delivery if the infant is Rh(D) positive

19-year-old woman, gravid 1, para 0, comes to the office at 28 weeks for a routine [renatal visit. Her prenatal course was complicated by vaginal bleeding at 8 weeks gestation due to a sub chronic hematoma. At that time, lab eval showed that sheas Rh(D) negative & she received anti-D immune globulin. The vaginal bleeding resolved & the patient has had no other problems during this pregnancy. Lab eval today shows a negative anti-D antibody screen. The Rh(D) status of the father of the child is unknown. Which of the following is the best next step concerning Rh(D) alloimmunization in this patient? A. Anti-D immune globulin should be administered now & postpartum B. anti-D globulin should be administered only postpartum C. no additional prophylaxis is indicated as she previously received anti-D immune globulin D. no prophylaxis is indicated as the father's status is unknown E. prophylaxis is not indicated as anti-D antibody screening is negative

C. flagellated, motile organisms Trichomonas vaginalis is a common cause of vaginitis & typically presents with a malodorous, thin, frothy, yellow-green vaginal d/c with an elevated pH (>4.5). Wet mount microscopy reveals motile, flagellated ovoid protozoa.

20-year-old woman presents due to 3 days of malodorous vaginal d/c & severe vulvar pruritus. She is sexually active & had a copper-containing IUD placed for contraception last year. She denies chronic medical conditions & has had no surgeries. VS are normal. Pelvic exam shows a thing, frothy & green vaginal d/c with marked vulvar & vaginal erythema. Vaginal d/c pH is 5.5. The IUD strings are visualized & protrude through the cervical os. Microscopic exam of this patient's vaginal d/c will most likely reveal which of the following? A. clue cells B. filamentous, gram-positive bacilli C. flagellated, motile organisms D. multinucleated giant cells E. pseudohyphae

B. admit for magnesium sulfate & induction of labor patients with preeclampsia with severe features at 34 weeks gestation require magnesium sulfate for seizure prophylaxis, antihypertensive therapy to reduce stroke risk if severe-range HTN is present, & delivery

20-year-old woman, gravid 1, para 0, at 34 weeks gestation presents to the office for a routine prenatal visit. She has had no headaches, changes in vision, or RUQ pain. Fetal movement is normal. The patient has no chronic medical conditions & the pregnancy has been uncomplicated. BP: 164/96 with a repeat reading of 166/94. Fundal height is 34cm. The cervix is 1cm dilated, 50% effaced, with the fetal vertex at -2 station. UA reveals 1+ protein. A non stress test is reactive. Lab results reveal a platelet count of 94,000mm3 & creatine level of 1.3 mg/dL. Which of the following is the best next step in management of this patient? A. admit for inpatient monitoring, bed rest & CCS B. admit for magnesium sulfate infusion & induction of labor C. follow up in 1 week with repeat lab & non stress testing D. prescribe labetalol & repeat BP in 48 hours E. prescribe nifedipine & perform 24-hour urine collection for total protein

A. Fetal crown-rump length 1st-trimester US measurement of fetal crown-rump length is the most accurate method of estimating gestational age & delivery date. Calculations based on the LMP & fundal heights are commonly used but less precise

22-year-old presents after a positive home pregnancy test. The patient thinks her LMP was "about 2 months ago" & has had no abnormal bleeding or pelvic pain since. PE reveals a gravity uterus that is palpable just above the symphysis pubis. Which of the following is the best method for estimating this patient's due date? A. Fetal crown-rump length B. Gestational sac diameter C. LMP calculation (Naegele rule) D. Quantitative b-hCG level E. Uterine fungal height measurement

E. Skin testing & PCN desensitization Treatment with PCN is required for all pregnant patients with syphilis to prevent fetal complications. Patients with PCN allergy should receive skin testing to confirm an IgE-mediated reaction. If the test is positive, patients require PCN desensitization prior to receiving treatment with IM PCN G Benzathine.

22-year-old primigravida woman at 8 weeks gestation comes to the office for her initial prenatal visit. She has no symptoms except mild nausea. The patient has a history of an allergic rxn to PCN; she developed a generalized rash with intense itching that responded to antihistamines & corticosteroid cream. Vital signs are normal. PE shows no abnormalities. The screening VDRL test is positive, as is the confirmatory fluorescent treponema antibody absorption test. Her HIV test is negative. Which of the following is the best next step in management of this patient? A. Azithromycin B. Doxycycline C. Erythromycin D. No treatment until delivery E. Skin testing & PCN desensitization

A. Acetaminophen acetaminophen is the 1st-line treatment in pregnant patients with migraines refractory to nonpharmacologic management. 2nd & 3rd line options include antiemetic, opioids & NSAIDs drugs (2nd trimester only)

25-year-old primigravida at 32 weeks gestation presents to ED due to severe headaches that began a month ago & now occur once or twice a week. they are unilateral, throbbing & associated with light sensitivity & nausea. The patient had similar headaches prior to pregnancy that usually improved with Ibuprofen & bed rest. VS & PE are normal. Which of the following is the most appropriate therapy for this patient? A. Acetaminophen B. Ergotamine C. Naproxen D. Sumatriptan E. Topiramate

D. Observation & reassurance Follicular ovarian cysts are typically smooth, thin-walled & < 10-cm. Rupture of these can occur with vigorous activity, resulting in sudden onset of pelvic pain. Hemodynamically stable patients with a follicular ovarian cyst are managed with observation & reassurance.

22-year-old woman comes to the ED with sudden-onset, right-sided pelvic pain that started while exercising. The patient has no associated nausea, vomiting or changes in bowel function. Her LMP was 3 weeks ago & she is not on contraception. Vitals are normal. The abdomen is slightly tender to palpation but there is no rebound or guarding. Pregnancy test is negative. Pelvic US shows a 4-cm, thin-walled ovarian cyst with Doppler flow & minimal free fluid in the posterior cup-de-sac. Hemoglobin is normal. Which of the following is the best next step in management of this patient? A. CA-125 level B. Diagnostic laparoscopy C. Empiric ABX D. Observation & reassurance E. Ovarian biopsy

B Infertility Endometriosis may present with pelvic pain, dyschezia, rectovaginal modularity & an ovarian mass (endometrioma). Endometriosis causes pelvic adhesions & inflammation, leading to infertility & chronic pelvic pain.

22-year-old woman comes to the office with RLQ pain for the past 3 weeks. The pain is exacerbated with bowel movements, but there is no associated constipation or Selena. She has no chronic medical conditions and has regular menstrual periods every 28-30 days. The patient is afebrile & BMI is 24 kg/m2. The abdomen is soft with normal bowel sounds. There is tenderness to deep palpation in the RLQ without rebound or guarding. The uterus is small, nontender & anteverted; there is no cervical discharge or cervical motion tenderness. The right adnexa is enlarged & tender. There is modularity & tenderness on rectovaginal exam. This patient is at increased risk for which of the following conditions? A. Cervical dysplasia B. Infertility C. Primary ovarian insufficiency D. Tube-ovarian abscess E. Uterine sarcoma

C. Levonorgestrel pills Women who are sexual assault victims are offered post exposure prophylaxis for STIs & emergency contraception. 1st-line emergency contraception is with Levonorgestrel pills (i.e. plan B) due to high efficacy rates & oral administration. IUD placement is contraindicated in patients who have acute cervicitis

22-year-old woman presents to ED after a sexual assault a few hours ago. The patient is not on contraception. Menses occur monthly & her LMP was 3 weeks ago. PE shows bruins & genital trauma. The cervix is edematous & friable. STI testing & forensic eval are completed. Urine hCG is negative. The patient asks for emergency contraception. Which of the following is the most appropriate management option for this patient? A. Combo OCPs B. Copper-containing IUD C. Levonorgestrel pills D. Methotrexate injection E. Misoprostol pills

E. Hyperemesis gravidarum Presents in 1st trimester with severe N/V & epigastric &/or RUQ pain. Mildly elevated AST and a hypochloremic metabolic alkalosis are common. Treatment includes inpatient admission for IV hydration, antiemetic therapy & correction of electrolyte abnormalities.

22-year-old woman, gravida 1 para 0, at 13 weeks gestation comes to the ED due to epigastric & RUQ pain over the past few hours. The patient has had persistent nausea & vomiting for several weeks. Temp is 36.7C (98F), BP is 120/80. Lab results include- Leukocytes: 9,000/mm3 (normal: 4,500-11,000) Platelets: 200,000/mm3 (normal: 150,000-400,000) Lipase: normal AST: 74 U/L (normal: 8-20) Which of the following is the most likely diagnosis? A. Acute cholangitis B. Acute fatty liver of pregnancy C. Acute pancreatitis D. HELLP syndrome E. Hyperemesis gravidarum

C. Handling & consumption of undercooked meats Congenital toxoplasmosis can lead to chorioretinitis, seizures & intellectual disability. Pregnant patients should avoid handling or eating undercooked meat to reduce the risk of toxoplasmosis infection & vertical transmission

23-year-old woke, gravitas 2 para 1, at 20 weeks gestation undergoes a routine prenatal US. The patient has no chronic medical conditions & takes a daily multivitamin. US reveals multiple intracranial calcification & bilateral cerebral ventriculomegaly, findings consistent with congenital toxoplasmosis. Which of the following historical factors resulted in this congenital infection? A. Absence of insect repellent or protective clothing while outdoors B. Consumption of unpasteurized milk & cheeses C. Handling & consumption of undercooked meats D. No adherence to vaccination schedule E. Unprotected intercourse with an infected partner

C. no contraindications HPV vaccination series is indicated for girls & women age 11-26 (but may be given up to age 45) & boys and men age 11-21 to reduce the risk of HPV-associated disease regardless of HIV status, prior incomplete vaccination or prior abnormal Pap tests.

23-year-old woman presents for a postpartum visit. The patient's Pap test at her initial prenatal visit revealed low-grade abnormalities and she was followed for chronic Hep B infection. Her prenatal course & delivery were otherwise uncomplicated. The patient smoked a pack of cigarettes a day prior to pregnancy but cut back to 2-3/day while pregnant and does not use EtOH or illicit drugs. She is breastfeeding her infant. The patient received all recommended vaccines during pregnancy but has not received the HPV vaccination series. Which of the following is a contraindication to the HPV vaccination series. Which of the following is a contraindication to the HPV vaccine in this patient? A. breastfeeding status B. Hep B infection C. No contraindication D. prior abnormal Pap test E. tobacco use

D. ovarian cyst rupture a ruptured ovarian cyst presents with acute onset of unilateral pelvic pain, often precipitation by strenuous activity or sexual intercourse. Pelvic US can confirm the diagnosis by showing free fluid in the pelvis from leaking cyst contents. Uncomplicated cyst rupture in hemodynamically stable patients can be managed conservatively.

23-year-old woman presents to the ED due to pelvic pain that started immediately after sexual intercourse 3 hours earlier. The pain is sharp, stabbing & has intensified significantly. Her LMP was 3 weeks ago. Temp is 36.7C ((*F). The abdomen is tender to palpation to the RLQ. Lab results: Hemoglobin: 12.2g/dL Platelets: 260,000/mm3 Leukocytes: 8,200/mm3 Urine pregnancy test is negative. Pelvic US shows a 4x5cm right cystic ovarian mass with a moderate amount of free fluid in the pelvis. Doppler velocimetry is normal. Which of the following is the most likely diagnosis? A. adnexal torsion B. endometriosis C. mittelschmerz D. ovarian cyst rupture E. tube-ovarian abscess

C. Neisseria gonorrhoeae Ruptured ectopic pregnancy can cause abdominal pain, vaginal bleeding & hemodynamic instability. Pelvic Inflammatory Disease (PID) is the most commonly caused by Neisseria gonorrhoeae or Chlamydia trachomatis & significantly increases the risk of ectopic pregnancy

23-year-old woman presents to the office for an eval of RLQ abdominal pain & bloody vaginal d/c. The symptoms began this morning & are worsening. She is sexually active with a male partner. They use condoms occasionally, but the patient is not sure whether her partner has other sexual partners. She was treated for a STI a few years ago. Her LMP was 5 weeks ago, and she has never been pregnant. BP: 112/70 while supine and 96/60 while standing. She is tachycardic & ill-appearing. Lab results are as follows: Hemoglobin: 12g/dL MCV: 82fL Platelets: 200,000/mm3 Leukocytes: 9,000/mm3 Urine B-hCG: positive Prior infection with which of the following microorganisms is most likely responsible for this patient's current condition? A. Gardnerella vaginalis B. HSV C. Neisseria gonorrhoeae D. Treponema pallidum E. Trichomonas vaginalis

D. Sheehan Syndrome is due to pituitary hypo perfusion that causes anterior pituitary infarction, usually following postpartum hemorrhage. The resulting prolactin deficiency can lead to failure of lactation. Other anterior pituitary hormone deficiencies (growth hormone, FSH, LH, TSH, ACTH) are also possible.

23-year-old woman presents to the office with 2 weeks of fatigue & excessive sleepiness. She delivered a healthy child 4 weeks ago, but the delivery was complicated by heavy bleeding. The patient has not lactated. She is otherwise healthy. Lab shows Hemoglobin of 9.8g/dL, TSH: 0.2uU/mL & low total and free T4 levels. Which of the following is the most likely diagnosis? A. Addison disease B. postpartum thyroiditis C. Prolactinoma D. Sheehan syndrome E. thyroid adenoma

D. multiple sex partners having multiple sex partners is associated with the highest increase in risk of PID. Other risk factors include being age 15-25, previously having PID, inconsistently using barrier contraception & having a partner with a STI.

23-year-old woman presents to the office with worsening fever, chills & lower abdominal pain for 3 days. She also has increasing malodorous vaginal d/c. The patient has a history of 2 episodes of PID that required hospitalization & IV ABX treatment. She has had 6 sexual partners over the last 6 months. The patient had an IUD placed a year ago and inconsistently uses barrier contraception. Temp is 38.3C (100.9F). Pelvic exam reveals cervical motion & bilateral adnexal tenderness as well as purulent d/c from the cervical os. The IUD strings are visible at the external cervical os. Nucleic acid amplification testing is positive for Neisseria gonorrhoeae. Which of the following is the strongest risk factor for this patient's condition? A. age < 25 B. inconsistent use of barrier C. intrauterine contraceptive device D. multiple sex partners E. previous episodes of PID

E. breastfeeding is recommended as the benefits outweigh the risks contraindications to breastfeeding include maternal HIV or active untreated TB, active viral skin lesions involving the nipple, maternal chemotherapy, illicit drugs use. &infant glactosemia. Mothers with Hep B can breastfeed if the infant has received Hep B immunoglobulin & initiation of the Hep B vaccine series

24-year-old woman is evaluated in the postpartum unit after a vaginal delivery of a term infant. The patient would like to breastfeed her baby. She has a history of chronic Hep B infection. The patient has smoked a half pack of cigarettes for 8 years & takes Buprenophrine daily for a history of opioid dependence. She did not use alcohol or illicit drugs during the pregnancy. Her medical and family medical history is otherwise unremarkable. Exam shows engorged breasts with no skin lesions. Which of the following is the best recommendation for this patient? A. breastfeeding is contraindicated due to maternal Hep B infection B. breastfeeding is contraindicated due to maternal tobacco use C. breastfeeding is contraindicated until the infant's newborn screen is normal D. breastfeeding is contraindicated until the mother is no longer on Buprenophrine E. breastfeeding is recommended as the benefits outweigh the risks

D. levonorgestrel-only pill Emergency contraception should be provided as soon as possible in patients who had unprotected intercourse. First-line options include the copper-containing IUD and Levonorgestrel pill.

24-year-old woman presents to the office for emergency contraception. The patient had unprotected intercourse with her boyfriend 2 nights ago after a condom broke. She is not using any form of contraception other than condoms and spermicide. Her LMP was 2 weeks ago. VS are normal. BMI: 26kg/m2. On spec exam, there is a mucopurulent d/c at the cervical os; no vaginal is noted. The uterus is small & mobile. There are no adnexal masses or tenderness. Urine pregnancy test is negative. Which of the following is the best next step in management of this patient? A. combo OCP B. copper-containing IUD C. etonogestrel subnormal implant D. levonorgestrel-only pill E. Methotrexate

A. Begin taking combined OCP OCPs decrease the risk of ovarian cancer by preventing ovulation & thereby decreasing damage & potential malignant transformation at the ovarian surface

25-year-old nulligravid woman comes to the ED b/c of a day-long history of pain in the LLQ. The pain is moderately controlled with ibuprofen. Her LMP was 2 weeks ago; menses are regular & last 4-5 days. She uses condoms for contraception. Her grandmother died from breast cancer at age 68 & her mother, diagnosed with ovarian cancer at age 60, tested negative for BRCA mutations. Transvaginal US reveals a 4-cm left Aden all mass that is simple & unilocular. Despite assurances that the cyst appears benign, the patient asks how she can reduce her risk of ovarian cancer. Which of the following is the best recommendation for ovarian cancer prevention in this patient? A. Begin taking combined OCP B. No recommendations because the patient is not high risk C. Recommend a prophylactic bilateral saplings-oophorectomy D. Recommend annual US & CA-125 testing E. Recommend copper-containing IUD

D. Fibrocystic changes fibrocystic breast changes are a common cause of cyclic breast pain in women of reproductive age. Classic clinical findings are diffusely nodular breasts with nonfocal tenderness

25-year-old woman presents for eval of breast pain that began 2 days ago. The pain is described as a constant soreness & the patient is unable to sleep prone b/c of it. The patient recently stopped taking her OCP b/c she is trying to conceive. Her LMP was 3 weeks ago; menses are regular & occur approximately every 28 days. On exam, the breast have no skin changes. Palpation reveals bilateral, nonfocal chest tenderness & diffuse, cordlike thickening of the breasts. Which of the following is the most likely diagnosis in this patient? A. Breast cyst B. Fat necrosis C. Fibroadenoma D. Fibrocystic changes E. Inflammatory breast cancer

D. incision and drainage a Bartholin gland abscess results from infection secondary to a blocker Bartholin gland duct. Patients typically have vulvar pain & a tender, fluctuant mass at the base of the labium magus that protrudes into the vagina. Treatment is with incision & drainage.

25-year-old woman presents with a painful bump in the genital area. The pain is especially bothersome when she sits, and she cannot have sexual intercourse due to excruciating pain. VS are normal. Exam shows a tender, 3-cm, fluctuant mass at the base of the right labium magus, bulging medially into the vagina. Spec exam is limited by patient discomfort, but no abnormal d/c is seen at the cervical os. The remainder of the pelvic exam is normal. Which of the following is the best treatment for this patient? A. Acyclovir B. Azithromycin + Ceftriaxone C. Bx of the lesion D. incision & drainage E. wide local excision

C. Oral contraceptives endometriosis can present with dysmenorrhea & deep dyspareunia. Symptomatic patients are offered empiric medical therapy with NSAIDs &/or combo oral contraceptives which treat inflammation & suppress ovarian stimulation of endometriosis

26-year-old nullliparous woman comes to the office due to severe pain during sexual intercourse. The pain is particularly unbearable with deep penetration, which has caused her to avoid sexual intimacy for the past few months. This has caused a significant strain in her relationship with her husband of 2 years. They have been using condoms with lubrication during intercourse, but this has not improved the pain. Menses occur every 30 days, are very painful & last for 5 days with moderate bleeding. A speculum can be inserted into vagina, but not fully opened due to patient discomfort. The uterus is immobile on bimanual exam. Which of the following is the most appropriate treatment for this patient's condition? A. Cognitive behavioral therapy B. Couples therapy & sexual education C. Oral contraceptives D. Pelvic floor physical therapy E. Vaginal estrogen therapy

C. Single-dose oral Metronidazole; express & discard breast milk for 24 hours Trichomoniasis is characterized by vulvovaginal pruritus & erythema; a frothy, yellow-green d/c; & punctate hemorrhages in the vagina or on the cervix ("strawberry cervix"). Single-dose oral Metronidazole is the preferred treatment. To avoid infant exposure, breast milk should be expressed & discarded for 24 hours after dose administration.

26-year-old woman comes to the office due to greenish d/c & vulvar pruritus that started several days ago. The patient had an uncomplicated vaginal delivery 2 months ago & is breastfeeding her infant exclusively. Temp is 36.7C (98F) & BP is 100/60. VUlvar edema & erythema are present. Pelvic exam reveals a nontender, anteverted uterus & no adnexal masses. Speculum exam shows punctate hemorrhages on the cervix. Which of the following is the most appropriate treatment for this patient? A. Oral Doxycycline for 7 days; d/c breastfeeding during treatment B. Single-dose oral Fluconazole; continue breastfeeding with no change C. Single-dose oral Metronidazole; express & discard breast milk for 24 hours D. Single-dose Metronidazole; replace breast milk with formula thereafter E. Vaginal Metronidazole for 5 days; continue breastfeeding with no change

B. Edinburgh postnatal depression scale postpartum depression has a high incidence & is often underreported. Therefore, all women (regardless of prior psychiatric history) require screening for postpartum depression (Edinburgh Postnatal Depression Scale).

26-year-old woman gravida 1 para 1, presents to the office for a 6-week postpartum checkup following a vaginal delivery at 28 weeks gestation for preeclampsia with severe features. The patient has no chronic medical conditions. She is breastfeeding & supplementing with pumped breast milk. She has not resumed her menses & is taking progestin-only pill for contraception. She says, "the baby is health which is great, but I haven't been sleeping much & have been very tired. My mom is coming to stay with us, so I'm hoping to get some rest soon." Temp is 36.7C (98F), BP: 126/80, HR: 68/min. BMI: 24kg/m2. Breast & pelvic exams are normal. Which of the following should be performed at this visit? A. 24-hour urine protein collection B. Edinburgh postnatal depression scale C. Glucose tolerance test D. thyroid function tests E. transvaginal US

D. maternal type 2 DM Shoulder dystocia, the impaction of the fetal shoulder behind the maternal pubic symphysis, presents as the retraction of the fetal head into the maternal perineum immediately after it delivers. Risk factors include conditions that cause fetal macrosomia such as maternal DM, maternal obesity & post-term (>42 weeks) gestation.

26-year-old woman, gravid 2 para 1, at 37 weeks gestation is admitted to the hospital in active labor. She receives an epidural & slowly progresses to complete cervical dilation. The patient pushes for 2 hours; the fetal head delivers, immediately retracts into the perineum & the anterior shoulder does not deliver. Which of the following is the greatest risk factor for the patient's clinical presentation? A. Fetal gestational age B. Maternal age at delivery C. Maternal chronic HTN D. Maternal type 2 DM E. Short interpregnancy interval

D. Oxytocin infusion 1st stage of labor is divided into 2 phases: the latent phase (0-6cm) & the active phase (> 6-10cm). Active phase protraction occurs when cervical dilation proceeds slower than expected (< 1cm every 2 hours). Contraction inadequacy is the most common etiology; management is with oxytocin augmentation.

27 year old woman, gravitate 1 para 0 at 40 weeks gestation presents to ED due to painful contractions for the past few hours. VS are normal. BMI: 42kg/m2. On admission, the cervix is 6 cm dilated & 100% effaced with the fetal head at -1 station. She received epidural analgesia and repeat cervical exam 2 hours is unchanged. Fetal HR monitoring shows moderate variability, multiple accelerations & no decelerations. The tocodynamometer shows contractions every 5-7 mins. The patient is diagnosed with active phase labor protraction. Which of the following is the best next step in management? A. C-section delivery B. Continue expectant management C. Operative vaginal delivery D. Oxytocin infusion E. vaginal misoprostol

A. continue yoga & jogging as tolerated throughout pregnancy Exercise for > 30 mins 5-7 days/week is recommended for all pregnant women who have no contraindications (preeclampsia). Contact sports & activities with a high fall risk are to be avoided due to the risk of fetal harm.

27-year-old female presents with questions about exercise after a positive home pregnancy test. The patient jogs 3-4 times a week & practices an hour of yoga 5 days a week. BMI: 24kg/m2. VS & PE are normal. Transvaginal US confirms an intrauterine pregnancy at 6 weeks gestation. Which of the following is the most appropriate advice for this patient? A. continue yoga & jogging as tolerated throughout pregnancy B. continue yoga as tolerated but d/c jogging C. d/c current exercise regimen & daily walking D. limit current exercise to < 30mins daily E. use pulse measurement to guide exercise intensity

A. colposcopy a high-grade squamous intraepithelial lesion Pap test result requires colposcopy & biopsy of cervical abnormalities due to a high risk of progression to cervical cancer

27-year-old woman presents for her annual exam. She has had no postcoital or intermenstrual bleeding, pelvic pain or abnormal vaginal d/c. She uses OCPs, is currently sexually active with 1 parter & has had 4 lifetime partners. Pelvic exam is normal. Pap test results reveal a high-grade squamous intraepithelial lesions. Which of the following is the best next step in management of this patient? A. colposcopy B. cryotherapy C. HPV co-testing D. repeat pap in 1 year E. trichloroacetic acid

E. Vaginal Clotrimazole Candida vulvovaginitis can present with thick, clumpy, non malodorous vaginal d/c, vulvovaginal pruritus, excoriation, edema & erythema; dyspareunia & dysuria. During pregnancy, 1st-line treatment options include vaginal Clotrimazole, Miconazole & Nystatin.

27-year-old woman, gravida 1 para 0, at 11 weeks gestation comes to the ED due to 2 days of a yellow, non-malodorous vaginal d/c. She has had no vaginal bleeding or contractions but has had increasing vulvar pruritus. BP is 128/78, HR is 86/min. Pelvic exam reveals no cervical motion or adnexal tenderness. On speculum exam, a thick clumpy d/c is seen coating the vaginal vault. The cervix is closed & no d/c from the cervical os is present. The pH of the fluid is 4. Which of the following is the most appropriate treatment for this patient? A. No treatment indicated B. Oral Azithromycin C. Oral Metronidazole D. Vaginal Clindamycin E. Vaginal Clotrimazole

B. Continue expectant management Fetal heart rate (FHR) monitoring identifies fetuses at risk for adverse outcomes due to hypoxia & metabolic acidosis. Category I FHR tracings (normal baseline, moderate variability, accelerations, and no decelerations) signify a normal fetal acid-base status and can be managed expectantly.

27-year-old woman, gravida 1 para 0, at 40 weeks gestation comes to the ED due to contractions & continuous leakage of fluid. Vitals are normal. Rupture of membranes is confirmed by speculum exam. The cervix is 2cm dilated & 30% effaced. Cephalic fetal presentation is confirmed. An external monitor is placed & the fetal heart rate tracing shows a baseline of 140/min, variability between 8 and 25/min and some accelerations. Which of the following is the best next step in management of this patient? A. administer oxygen B. Continue expectant management C. Perform cesarean delivery D. place fetal scalp electrode E. place intrauterine pressure catheter

A. Admit for inpatient administration of IV Ceftriaxone Pyelonephritis in pregnancy is associated with maternal (eg. acute respiratory distress syndrome) & obstetric (eg preterm labor) complications. Therefore, patients are managed with inpatient parenteral ABX until afebrile for 24-48 hours.

27-year-old woman, gravitate 1 para 0, at 29 weeks, gestation comes to the emergency department due to fever, mild nausea & 2 days of urinary frequency & urgency. She has no vomiting or abdominal pain. The patient was recently diagnosed with gestation DM. She had NKDA. On exam, there is a mild costovertebral angle tenderness. Leukocyte count is 14,800/mm3 with 88% neutrophils. UA results: specific gravity: 1.02 blood: - ketone: - leukocyte esterase: + Nitrites: + bacteria: moderate WBC: 25+/hpf RBC: 1-2/hpf A urine culture is obtained. Which of the following is the most appropriate management for this patient? A. Admit for inpatient administration of IV Ceftriaxone B. Admit for inpatient administration of IV Ciprofloxacin C. D/c on oral Cephalexin with next-day outpatient follow up D. D/c on oral Nitrofurantoin with next-day outpatient follow up E. Obtain renal US, if normal, d/c on oral Cephalexin

A. Decreased LES tone pregnant women often develop GERD due to elevated levels of estrogen & progesterone, which relax the smooth muscle of the LES. Later in pregnancy, GERD can also develop when the gravity uterus presses on the stomach & leads to an altered LES angle or increased gastric pressure

28-year-old woman at 32 weeks gestation is seen in the office for a routine prenatal visit. The patient has had burning, squeezing pain in the middle of her chest that lasts for minutes to hours at a time & usually occurs after meals. The pain is occasionally accompanied by small amounts of sour-tasting material in the back of her throat. The patient does not have any abdominal pain, emesis or blood in stool. Her pregnancy has been complicated by gestational diabetes mellitus, which has been well controlled with diet & exercise. VS are normal. Fundal height is 32 cm & fetal heart tones are normal. Which of the following is the most likely cause of this patient's symptoms? A. Decreased LES tone B. Delayed gastric emptying C. Diffuse spasm of the esophageal smooth muscle D. H. Pylori infection E. Increased gastric acid production

B. Endometrial glands & stroma Endometriosis is ectopic implants of endometrial glands & storm in the abdominopelvic cavity. Symptoms are based on implant location Bladder implants may cause suprapubic tenderness & dysuria. Rectovaginal implants may cause dyschezia, rectovaginal modularity & pelvic fibrosis, as evidenced by an immobile uterus on exam.

28-year-old woman comes to the office due to discomfort during urination increasing pain with the passage of stooging, particularly during menses. Bowel movements occur daily & are soft & not watery. Her symptoms have no improves with the elimination of caffeine & alcohol from her diet. The patient has also had intermittent abdominal pain for the past 7 months. Exam shows suprapubic tenderness & an immobile, retroverted uterus. Rectovaginal exam reveals modularity in the posterior cup-de-sac. Surgical biopsy of the pelvic nodules would most likely show which of the following? A. Colorecal hamartoma B. Endometrial glands & stroma C. Endometrial intraepithelial neoplasia D. Serous epithelial ovarian carcinoma E. Uterine myometrium

B. chronic suppressive antiviral therapy HSV transmission can occur with asymptomatic viral shedding & during prodromal phases & active lesions. The most effective strategy for reducing transmission is daily suppressive therapy

28-year-old woman presents to the office for consultation. The patient has a Hx of recurrent genital HSV2 infections but has no symptoms at present. She has 3 or 4 symptomatic episodes per year. The patient has no other sexually transmitted infections or medical problems & takes no meds. She has a new male sexual partner who is HSV-seronegative. Which of the following is the most effective strategy for decreasing this patient's risk of transmitting HSV to her partner? A. abstinence for 2 weeks after an outbreak B. chronic suppressive antiviral therapy C. circumcision of the male partner D. condom use during prodromes & outbreaks E. prompt antiviral treatment of outbreaks

E. Serum prolactin level Secondary amenorrhea is amenorrhea for > 3 months in women with previously regular menses (or > 6 months in women with previously irregular menses). Initial eval includes a pregnancy test, followed by serum prolactin, TSH & FSH level testing.

29-year-old nulligravid woman comes to the office bc she has not menstruated since stopping her birth control. Menarche was at age 14, and she had irregular, heavy menstrual periods from age 14-16. The patient then took OCP until 8 months ago. Since stopping her med, she has had more frequent headaches & some acne but has had no changes in weight, pelvic pain or abnormal loss or growth of hair. BMI is 22kg/m2. A fe open comedones are seen on the forehead. The remainder of the PE, including pelvic exam is normal. Urine pregnancy test is negative & pelvic US is normal. Which of the following is the best next step in eval of this patient? A. Karyotype analysis B. MRI of pituitary C. Nor further eval D. Serum 17-hydroxyprogesterone level E. Serum prolactin level

B. Infundibulopelvic (suspensory) ligament Ovarian torsion typically involves twisting of the infundibulopelvic ligament (or suspensory ligament of the ovary), often due to the weight of a large adnexal mass. The resulting occlusion of the blood & nerve supply to the ovary results in severe, acute pelvic pain & ovarian ischemia.

29-year-old nulliparous woman comes to the ED with right-sided pelvic pain. The pain is sharp & came on suddenly while she was shoveling snow this morning. It is accompanied by nausea & is worse with movement. The patient's LMP was 2 weeks ago. Temp is 37.8C (100F), BP is 100/60, HR is 92/min. Pelvic US reveals a normal-sized uterus & left ovary & a right adnexal mass measuring 11cm with no blood flow to the ovary. Urine B-hCG is negative. The patient's condition most likely involves pathology in which of the following structures? A. Broad ligament B. Infundibulopelvic (suspensory) ligament C. Mesosalpinx D. Round ligament E. Uterosacral ligament

A. anti-D immune globulin Complete abortions are a pregnancy loss at < 20 weeks gestation with spontaneous evacuation of products of conception. In Rh (D)-negative women, anti-D immune globulin is administered after a spontaneous (hydrops fettles) in future pregnancies.

30-year-old woman, gravid 3 para 2, at 9 weeks gestation presents to the ED due to abdominal cramping & heavy vaginal bleeding. Patient has passed large fist-sized clots for the last 4 hours. On arrival, patient passes a white mass covered in blood. Her pain resolves immediately & the bleeding improves. Her first prenatal visit was a week ago & an US at that time showed a viable intrauterine gestation. Temp is 37.2C (99F), BP: 100/70 & HR: 94/min. On pelvic exam, the cervix is closed & uterus is nontender. Hemoglobin is 8.8g/dL & blood type is AB, Rh negative. Which of the following is the best next step in management of this patient? A. Anti-D immune globulin B. Ceftriaxone + Doxycycline C. Misoprostol D. Oxytocin E. Suction curettage

C. inevitable abortion An inevitable abortion presents with vaginal bleeding, lower abdominal pain & a dilated cervix without expulsion of the products of conception. US typically reveals a nonviable gestation in the lower uterine segment.

31-year-old primigravida at 7 weeks gestation is brought to the ED due to vaginal bleeding & lower abdominal pain. The bleeding began 4 hours earlier, & the patient has saturated 3 pads in that time. Her abdominal cramping has become progressively more severe. She had a loop electrode excision procedure for high-grade cervical intraepithelial neoplasia at age 29. Temp is 37C (98.7F), BP: 100/76 & HR: 112/min. Pelvic exam reveals blood clots in the vaginal vault & active bleeding from a dilated cervix. A bimanual exam demonstrates a 6-week-size tender uterus. US reveals a gestational sac in the lower segment of the uterus & a simple cyst in the right ovary. Which of the following is the most likely diagnosis of this patient? A. cervical insufficiency B. ectopic pregnancy C. inevitable abortion D. missed abortion E. threatened abortion

A. Abruptio placentae placental detachment before fetal delivery— causes vaginal bleeding, painful contractions, uterine rigidity & fetal decelerations. Maternal HTN (eg gestational HTN, preeclampsia) increases the risk of placental abruption.

32-year-old primigravida at 34 weeks gestation presents to ED due to heavy vaginal bleeding & painful contractions. The patient's pregnancy has been complicated by gestational HTN. BP: 159/94 & HR: 100/min. The fetal HR tracing shows recurrent decelerations; tocadynamometry shows contractions every 1-2 mins. The uterus is firm & tender. On pelvic exam, the cervix is visibly 4 cm dilated & there are large blood clots in the vaginal vault. Which of the following is the most likely diagnosis in this patient? A. Abruptly placentae B. Placenta accreta C. Placenta previa D. Uterine rupture E. Vasa Previa

C. Human papillomavirus Cervical cancer, an AIDs-defining illness in patients with HIV is caused by high-risk strains (types 16 & 18) of HPV. Cervical cancer can present with postcoital bleeding, a painless ulcerative lesion & painless inguinal lymphadenopathy.

32-year-old woman comes to the office due to postcoital bleeding. She has monthly menstrual cycles with 4 days of light, vaginal bleeding, but for the past 2 months, she has also had postcoital bleeding. 3 years ago, the patient was diagnosed with HIV, but she stopped taking antiretroviral therapy due to ADE & has not seen a provider since. On exam, there is an ulcerative lesion on the posterior aspect of the cervix that bleeds with contact. The rest of the cervix has no lesions, & no endocervical d/c is present. The left inguinal lymph nodes are enlarged & nontender. Which of the following pathogens is the most likely cause of this patient's presentation? A. Chlamydia trachomatis B. Gardnerella vaginalis C. Human papillomavirus D. Treponema pallidum E. Trichomonas vaginalis

A. Prescribe insulin therapy Initial management is dietary modifications to achieve euglycemia. Blood glucose levels should be measured with fasting and 1- & 2- hr postprandial. Target levels are: fasting <95 1hr postprandial < 140 2hr postprandial < 120 Patients who remain hyperglycemic (>50% of readings are above target levels) despite nutritional therapy require pharmacotherapy. Insulin does NOT cross the placenta & is 1st-line therapy for GDM unresponsive to diet.

32-year-old woman comes to the office for a follow-up appointment at 32 weeks gestation. She was diagnosed with gestational diabetes at 28 weeks gestation. The patient has since been eating 3 small, healthy meals & 2 snacks daily. BP is 126/80 & HR is 76/min. Blood glucose levels over the last week Day 1: Fasting= 112; breakfast 2-hr postprandial= 151; lunch 2-hr postprandial= 147; dinner 2-hr postprandial= 141 Day 2: fasting= 115; breakfast 2hr postprandial= 143; lunch 2 hr postprandial= 158; dinner 2 hr postprandial= 150 Day 3: fasting = 121; breakfast 2 hr postprandial= 162; lunch 2hr postprandial= 158; dinner 2 hr postprandial= 153 Day 4: fasting= 110; breakfast 2 hr postprandial= 161; lunch 2hr postprandial= 154; dinner 2 hr postprandial= 155 Which of the following is the best next step in management of this patient? A. Prescribe insulin therapy B. Prescribe Pioglitazone C. Reassure & observe D. Restrict caloric intake E. Start weight-loss program

E. Umbilical cord compression A variable deceleration is an abrupt decrease in FHR > 15/min from baseline with a duration of > 15 secs to < 2 minutes. A common etiology of variable decelerations is umbilical cord compression from acute amniotic fluid loss (rupture of membranes).

32-year-old woman, gravida 2, para 1, at 39 weeks gestation is admitted for active labor. She has regular fetal movements. Temp is 37.2C (99F), BP is 118/60, HR is 86/min. The cervix is 7cm dilated & 80% effaced, and the fetal head is at -3 station. Spontaneous rupture of membranes occurs and clear fluid is seen. The FHR tracing then shows multiple abrupt decelerations that nadir 20/min from the baseline. Which of the following is the most likely etiology of this patient's FHR tracing? A. Fetal growth restriction B. Fetal head compression C. Intrauterine infection D. Placental insufficiency E. Umbilical cord compression

A. Candida albicans Candida vulvovaginitis is caused by overgrowth of Candida albicans & typically presents with vulvovaginal erythema, vaginal pruritus & d/c with a normal vaginal pH (3.8-4.5). Treatment options include oral (Fluconazole) & intravaginal (Clotrimazole) antifungals.

32-year-old woman presents due to several days of vaginal itching & d/c. She also has pain with sexual intercourse. She denies fever, chills, abdominal pain, urinary frequency or dysuria. In the past years she has had 2 new male sexual partners. She takes OCP & uses condoms on most occasions. Patient denies any chronic medical conditions & denies prior surgeries. Denies any other medications. Her LMP was 3 weeks ago. Temp is 36.7C (98F) & BP: 100/60. Pelvic exam shows vulvar & vaginal erythema & a small amount of vaginal d/c with a pH of 4. Which of the following is the most causative organisms? A. Candida albicans B. Chlamydia trachomatis C. Gardnerella vaginalis D. Neisseria gonorrheae E. Trichomonas vaginalis

A. Asherman Syndrome Asherman Syndrome, the formation of intrauterine adhesions, often occurs following intrauterine surgery (suction curettage). Patients with Asherman Syndrome typically have amenorrhea that does not respond to a progesterone challenge due to obliteration of the endometrium

32-year-old woman presents to the office for an eval of absent menses. She had a vaginal delivery 4 months ago, and she has not had a menstrual period since delivery. Her postpartum course was complicated by a postpartum hemorrhage, requiring blood transfusion & emergency suction & sharp curettage. Prior to this pregnancy, she had regular, monthly menstrual cycles. The patient has had increased fatigue since returning to work & is bottle-feeding. She has no headaches, galactorrhea or hot flushes. VS are normal. BMI: 31kg/m2. The uterus is small & anteverted and there are bilateral small, nontender ovaries. Urine pregnancy test is negative. FSH & TSH levels are normal. Which of the following is the most likely diagnosis in this patient? A. Asherman Syndrome B. Gestational trophoblastic disease C. Postpartum thyroiditis D. Primary ovarian insufficiency E. Sheehan Syndrome

B. cervical insufficiency cervical insufficiency refers to a structural weakness of the cervix. Diagnosis can be made retrospectively based on a history of > 2 spontaneous painless second-trimester losses or on presentation of painless advanced cervical dilation at < 24 weeks in the current pregnancy

32-year-old woman, gravid 1 para 0, at 19 weeks gestation comes to the ED due to vaginal pressure & spotting. Over the last few days, the patient has had clear vaginal d/c. Today she has had increasing pelvic pressure & has noticed blood spots on her underwear. She has had no cramping or pelvic pain; fetal movement has been normal. VS are normal. Fetal heart rate is 145/min. Speculum exam shows amniotic membranes bulging at the vaginal Introits. There is no blood or pooling of the fluid in the vaginal vault. Nitrazine paper does not turn blue on application of the vaginal d/c. The cervix is 3cm dilated & 100% effaced on digital exam. Which of the following is the most likely diagnosis in this patient? A. acute cervicitis B. cervical insufficiency C. preterm labor D. previable premature rupture of membranes E. threatened abortion

E. Progestin-releasing IUD patients with PCOS typically have irregular menses, androgen excess & polycystic ovaries on US. The unopposed estrogen excess caused by a ovulation results in an increased risk for endometrial hyperplasia or cancer. The endometrium can be protected by use of a progestin-containing IUD, which simultaneously provided contraception.

33-year-old woman comes to the office for a contraception prescription. The patient previously used OCPs for menstrual regulation & acne treatment, but she frequently forgot to take them, resulting in an unintended pregnancy 2 years ago. During her pregnancy, she gained 36.3kg (80lb) & has been unable to return to her pre-pregnancy weight. Her postpartum course was complicated by severe postpartum depression & she doesn't want to become pregnant again. Her menses occur every 45-90 days with 5-6 days of heaving bleeding & passage of large clots. BMI: 33kg/m2. Physical exam is normal. Serum chemistries, prolactin & TSH levels are normal. Urine pregnancy test is negative. Pelvic US reveals a small anteverted uterus & multifollicular ovaries bilaterally. Which of the following is the best contraceptive option for this patient? A. Copper-containing IUD B. Cyclic progestin-only pills C. Laparoscopic tubal ligation D. Low-dose combo OCPs E. Progestin-releasing IUD

D. insufficient ABX therapy PID is most frequently caused by Neisseria gonorrhoeae & Chlamydia trachomatis. Severe or inadequately treated PID can result in Fallopian tube scarring, which in turn can lead to infertility. Treatment is dual therapy with a 3rd-generation Cephalosporin (Ceftriaxone) plus Azithromycin or Doxycycline

33-year-old woman presents to a fertility clinic for eval. She has been attempting to conceive for 1 year with her current partner of 5 years without success. Her monogamous partner has been found to have adequate sperm motility, volume & count. The patient used OCPs for 15 years but stopped a year ago. 7 years ago, she received Ceftriaxone for fever, abdominal pain & vaginal d/c. Which of the following is the most likely major contributing factor to her inability to conceive? A. advanced maternal age B. congenital absence of the vas deferens C. inadequate time for attempted conception D. insufficient ABX therapy E. long-term OCP use

E. pelvic US with Doppler Ovarian torsion typically occurs in reproductive-age women & presents with sudden onset of unilateral pelvic pain & a tender adnexal mass. First-line is a pelvic US with color Doppler, which typically shows an enlarged, edematous ovary with decreased blood flow.

33-year-old woman presents to the ED due to abdominal pain. The pain began 2 weeks ago & was initially mild, episodic, confined to RLQ & resolved spontaneously without intervention. For the past 6 hours, the pain has been severe, constant & radiating to the right flank. The pain is now accompanied by constant nausea & multiple episodes of emesis. Temp is 38C (100.4F), BP: 130/80, HR: 83/min. Abdominal exam shows tenderness & guarding in the RLQ. Pelvic exam shows a small, nontender, retroverted uterus & a tender right adnexal mass. Speculum exam is normal. Urine pregnancy test is negative. Which of the following is the best next step in management of this patient? A. Chlamydia & gonorrhea nucleic acid amplification testing B. CT-guided drainage of the adnexal mass C. IV ABZ D. pain control & outpatient follow-up with gynecologist E. pelvic US with Doppler

E. Oral glucose challenge test During the second and third trimesters, placental hormones creates increased maternal insulin resistance & can result in pathologic maternal hyperglycemia. All women at 24-28 weeks gestation should undergo routine screening for gestational DM.

33-year-old woman, gravida 2 para 1, comes to the clinic for routine prenatal visit at 26 weeks gestation. Her pregnancy has been uncomplicated. The patient's BMI was 23 kg/m2 at her initial prenatal visit & her lab results: Hematocrit: 34% HIV-1 antibody: negative Rapid plasma reagin: not detected ABO/Rh type: A positive Urine culture: negative Chlamydia trachomatis PCR: negative The patient has no chronic medical problems & takes only prenatal vitamins. BP is 120/80 & HR: 65/min. PE shows minimal pedal edema. Fundal height is 27cm & fetal heart rate is 140/min. Which of the following tests should be obtained at this visit? A. 24-hour protein collection B. Chlamydia trachomatis cervical polymerase chain reaction C. Clean catch urine culture D. Group B Streptococcus rectovaginal culture E. Oral glucose challenge test

E. primary ovarian insufficiency (POI) POI occurs due to accelerated ovarian follicle depletion in women age < 40, resulting in secondary amenorrhea, elevated FSH & low estrogen levels

34-year-old woman presents for an eval of amenorrhea. Last year the patient had a missed abortion that was surgically managed with suction curettage. Her menstrual periods are initially heavy after the procedure, but the bleeding became increasingly lighter. Now the patient has not had a menstrual period in 6 months. She previously had regular menses. Patient has had no weight loss or changes in bowel movements. VS are normal. BMI: 22kg/m2. Pelvic exam shows a small, mobile uterus with no adnexal masses. Pregnancy test is negative. TSH is 3uU/mL and FSH is 52mU/mL (normal: < 40mU/mL). A progesterone challenge is performed & the patient has no withdrawal bleeding. Which of the following is the most likely diagnosis in this patient? A. functional hypothalamic amenorrhea B. hyperthyroidism-induced amenorrhea C. intrauterine adhesions D. polycystic ovary syndrome E. primary ovarian insufficiency

C. menstrual bleeding pattern Unexplained, abnormal vaginal bleeding is a contraindication to IUD insertion as it may indicate an underlying condition (endometrial cancer, sexually transmitted infection). Patients with unexplained vaginal bleeding require further eval (endometrial biopsy).

34-year-old woman presents to discuss the progestin-releasing IUD. The patient previously used condoms for contraception but is interested in starting a more reliable method. Her menses occur every 8-12 weeks with heavy vaginal bleeding & passage of clots & she has intermittent light spotting in between. She has no chronic medical conditions. The patient was treated for Trichomonas vaginitis at age 16. Her maternal grandmother had metastatic breast cancer in her 70s. BMI is 44kg/m2. Pelvic exam shows nulliparous cervix & a small nontender uterus. Urine pregnancy test is negative. Which of the following is this patient's contraindication to a progestin-releasing IUD? A. age B. family history C. menstrual bleeding pattern D. nulliparity E. prior sexually transmitted infection

D. no screening recommended women who have sex with women have risk factors for ovarian cancer (low parity, low contraceptive use); however despite these risk factors, they are still considered to be at average risk. Ovarian cancer screening in low- or average-risk women is not indicated.

35-year-old woman comes to the office to discuss her risk of ovarian cancer. The patient recently read that women who have sex with women might be at risk of ovarian cancer. She has had only female sex partners. The patient has no appetite changes, abdominal dissension, pelvic pain or abnormal vaginal bleeding. She is nulliparous & has never been on contraception. The patient has no chronic medical conditions & has had no prior surgeries. She takes a daily multivitamin & occasional ibuprofen at the beginning of her menstrual periods. Family history is significant for a maternal aunt with post menopausal breast cancer. Which of the following is the most appropriate ovarian cancer screening guideline for this patient? A. annual CA-125 levels B. annual transvaginal US C. biannual pelvic exams D. no screening recommended E. referral for BRCA testing

A. laparoscopy Patients with an ectopic pregnancy who have medical contraindications to methotrexate (medication non adherence, immunosuppression, anemia, thrombocytopenia) or who are hemodynamically unstable require surgical management.

35-year-old woman presents to the ED due to vaginal spotting. The patient's LMP was 6 weeks ago. She is HIV positive & non adherent to her antiretroviral Tx; she has had 2 ectopic pregnancies. VS are stable. Pelvic exam reveals a right adnexal mass. Hemoglobin is 8g/dL & platelet count is 60,000/mm3. B-hCG is 6215 IU/L. Pelvic US shows a 4.6cm complex adnexal mass with no pelvic free fluid. Which of the following is the best next step in management of this patient? A. laparoscopy B. methotrexate C. misoprostol D. repeat B-hCG level in 48 hours E. suction curettage

E. PCN Intrapartum ABX prophylaxis with PCN is indicated for women at risk for early-onset neonatal group B Streptococcus (GBS) disease. At-risk women include those with a positive GBS culture at 35-37 weeks gestation; those with a previous child with GBS disease; and those with an unknown GBS status plus an additional risk factor for early-onset GBS disease (intrapartum fever, rupture of membranes > 18 hours, preterm delivery).

35-year-old woman, gravid 1 para 0, at 40 weeks gestation comes to the labor & delivery unit for worsening contractions. The patient had spontaneous rupture of membranes 22 hours ago. Her last prenatal visit was at 32 weeks gestation. Temp is 37.2C (99F) & BP: 122/78. Fetal heart rate tracing shows a baseline heart rate of 160/min, multiple accelerations & no decelerations. Tocometry shows contractions every 3 mins. The cervix is 6cm dilated & 80% effaced, with the fetal vertex at -1 station. The amniotic fluid is meconium stained. Her group B Streptococcus status is unknown. Which of the following is the best next step in management of this patient? A. Amnioinfusion B. Cesarean delivery C. Magnesium D. Oxytocin E. PCN

A. Fetal aneuploidy testing Pregnant patients age > 35 are screened for fetal aneuploidy (trisomy 21, 18, 13) with cell-free fetal DNA testing, a highly sensitive & specific test that sequences fetal DNA in the maternal circulation. Testing can be performed at > 10 weeks gestation.

35-year-old woman, gravid 2 para 1, presents to the office for an initial prenatal visit. She has had no vaginal bleeding or cramping. LMP was 12 weeks ago. She denies chronic medical conditions & her previous pregnancy was uncomplicated. VS are normal. BMI: 23kg/m2. Pelvic US reveals a 12-week intrauterine gestation with normal fetal cardiac activity. In addition to routine prenatal lab tests, which of the following tests is recommended for this patient at this time? A. fetal aneuploidy testing B. Group B Strep rectovaginal culture C. Hemoglobin A1c D. Hep C antibody E. Wet mount microscopy

D. Prescribe ABX therapy & continue breastfeeding Lactational mastitis presents with flulike symptoms, focal unilateral breast pain with surround erythema & induration & axillary lymphadenopathy. Treatment includes ABX therapy against MRSA Staph aureus (Dicloxacillin, Cephalexin), analgesics & continued breastfeeding.

36-year-old woman, gravida 2 para 2, comes to the office due to 2 days of right breast pain. She has also had fever, chills, muscle aches, & fatigue. The patient had a spontaneous vaginal delivery 6 weeks ago and has been breastfeeding her infant. Her husband has been feeding their baby pumped breast milk so that she can rest at night. The patient has a no chronic medical conditions & takes a daily multivitamin. Temp is 38.8C (101.*F), BP is 110/60, HR is 84/min. PE shows a 5-cm area of erythema, induration, and tenderness at the upper outer quadrant of the right breast as well as right axillary lymphadenopathy. There is no fluctuant. The left breast has no abnormalities. Which of the following is the best next step in management of this patient? A. D/c breastfeeding & prescribe ABX therapy B. Order mammography & core needle biopsy C. Perform I&D and prescribed ABX therapy D. Prescribe ABX & continue breastfeeding E. Recommend NSAIDs & warm compresses only

D. Labetalol Gestational HTN is new-onset HTN (>140/>90) at >20 weeks gestation with no proteinuria or other signs of end-organ damage (HA, elevated transaminases). It is due to dysfunctional lateral & placental vascularity, causing tissue hypo perfusion & ischemia. It increases the res of maternal (stroke, seizure), fetal (growth restriction) & obstetric (abruptio placentae) complications. 1st line therapy, based on safety profiles in pregnancy & efficacy, include: beta blockers (Labetalol), Hydralazine & CCB (Nifedipine)

37-year-old woman, gravid 1 para 0, comes to the office for a routine prenatal visit at 28 weeks gestation. The patient has had no HA, visual disturbances or RUQ pain. Fetal movement is normal. BP is 164/96 & repeat BP is 150/110. There is 1+ pedal edema over the shins; the remainder of the PE is unremarkable. UA shows no proteinuria. CBC, creatinine & LFTs are normal. Which of the following is the most appropriate initial treatment for BP control in this patient? A. Clonidine B. Enalapril C. HCTZ D. Labetalol E. No treatment indicated

E. suction curettage a hydatidiform mole can present with pelvic pressure; a uterine size that is larger than expected for gestational age; & an US showing an enlarged uterus containing a heterogeneous mass composed to cystic structures. Management includes uterine evacuation with suction curettage

37-year-old woman, gravid 2 para 1, comes to the office to establish prenatal care. She is at 10 weeks gestation by a sure, regular last menstrual period. The patient has had some brown vaginal discharge & pelvic pressure over the past several days but no frank blood or abdominal pain BP: 130/80, HR: 92/min. A 14-week- sized uterus & small, bilateral ovaries are palpated on bimanual exam. Pelvic US reveals an enlarged uterus filled with heterogeneous mass composed of cystic structures. A urine pregnancy test is positive. Which of the following is the best next step in management of this patient? A. administer misoprostol B. diagnostic laparoscopy C. observation & serial B-hCG levels D. repeat US in 1 week E. suction curettage

C. Endometrial biopsy women age < 45 with AUB who have failed medical management (i.e. OCP) require eval for endometrial hyperplasia/cancer with an endometrial biopsy

38-year-old nulligravid woman presents due to persistent AUB. The patient's menstrual periods previously occurred monthly & consisted of 4 days of moderate bleeding & light cramping. However, for the past 8 months, she has had intermenstual spotting & bleeding that have occurred at varying intervals & last 3-7 days. She was started on combo OCPs 4 months ago, which has not improved the bleeding pattern. Temp: 37.2C (99F); BP: 126/76; HR: 86/min; BMI: 29kg/m2. Speculum exam shows dark red blood in the posterior vaginal vault but no cervical or vaginal lesions. Remainder of the pelvic exam is normal. Lab results: hemoglobin: 12.2g/dL (Normal: 12-16) prolactin: 5 ng/mL (normal: < 20) TSH: 1.8 uU/mL (normal: 0.5-5.0) Urine hCG is negative. Pelvic US shows an anteverted uterus & no adnexal masses. Which of the following is the best next step in the management of this patient? A. Coagulation studies B. Endometrial ablation C. Endometrial biopsy D. Hysterosalpingogram E. Progesterone withdrawal test

D. Pap & HPV cotesting at 1 and 2 years Patients who undergo cervical conization for cervical intraepithelial neoplasia (CIN) 3 and have surgical margins free of disease remain at risk for recurrent dysplasia and cancer. Therefore, these patients require more frequent cervical cancer screening sixth repeat Pap & HPV cotesting at 1 and 2 years.

38-year-old previously healthy woman comes to the office for follow-up of her first abnormal Pap test result. Last month, Pap testing revealed a high-grade squamous epithelial lesion. She then underwent a colposcopy, which confirmed high-grade cervical intraepithelial neoplasia (CIN) 3. Cervical ionization was performed and the pathology report showed CIN 3, with all surgical margins free of disease. Which of the following is the best next step in management of this patient? A. Endocervical curettage B. Hysterectomy C. No further pap testing D. Pap & HPV cotesting at 1 and 2 years E. Pap testing alone in 3 years

C. Uterine atony The most common cause of primary postpartum hemorrhage is uterine atony, which results from failure of the uterus to contract & compress the placental site blood vessels. Uterine atony presents with profuse vaginal bleeding & a soft ("boggy") & enlarged uterus. Risk factors include prolonged labor, induction of labor, operative vaginal delivery & fetal weight > 4000g (8.8lb).

39-year-old woman, gravida 1 para 0, at 38 weeks gestation comes to the hospital for induction of labor. After a prolonged labor, she delivers a 4.1kg (9lb) healthy boy via forceps-assisted vaginal delivery. During delivery of the placenta, excessive traction is placed on the cord, causing it to avulse & the placenta is manually extracted in pieces. An US after the extraction shows no placental tissue. 60 mins after delivery, the patient skas her perineal pad. During bimanual exam, 300mL of clotted blood is expressed from the lower uterine segment. The uterus is soft & 4 cm above the umbilicus after expression of the clots. Which of the following is the most likely cause of this patient's bleeding? A. Coagulopathy B. Retained placenta C. Uterine atony D. Uterine inversion E. Uterine rupture

E. Suction curettage Septic abortion most commonly occurs after an unsterile &/or incomplete procedure for an elective abortion; it presents with fever, heavy vaginal bleeding, purulent d/c & uterine tenderness. Septic abortion is a medical emergency that requires prompt treatment with broad-spectrum ABX & surgical evacuation of the uterus (suction curettage).

39-year-old woman, gravida 4 para 3, comes to the ED after 2 days of fever, heavy vaginal bleeding & lower abdominal pain. 6 days ago, the patient had an elective termination of pregnancy at 8 weeks gestation at an outside clinic. Since then, she has had bleeding that has become increasingly heavy & malodorous and is now soaking through a pad every 2-3 hours. Temp is 39.8C (103.6F), BP is 100/65, HR is 108/min and RR are 16/min. The patient has lower abdominal tenderness. Pelvic exam shows copious purulent, bloody d/c from the cervical os. The uterus is tender, anteverted & soft. Transvaginal US reveals retained products of conception. The adnexa are normal bilaterally. Broad-spectrum intravenous ABX are initated. Which of the following is the best next step in management of this patient? A. Hysterectomy B. Methotrexate C. Misoprostol D. Observation E. Suction curettage

B. missed abortion Patients with missed abortions may be asymptomatic or have loss of pregnancy symptoms. Diagnosis is confirmed with falling B-hCG levels & US findings of a nonviable pregnancy (empty gestational sac)

40-year-ol woman, gravid 1 para 0, comes to the office for an initial prenatal visit after a positive home pregnancy test. She has had no abdominal pain or vaginal bleeding. She has a long history of infertility & has tried to conceive for the last 5 years. The patient is at 6 weeks gestation based on her LMP. VS are normal. US reveals on ovoid gestational sac, without a yolk sac or fetal pole. No adnexal masses are seen. Serum B-hCG level is 2,000IU/L. Serum B-hCG level repeated 48 hours later is 1,800IU/L. Which of the following is the most likely diagnosis in this patient? A. complete hydatidiform mole B. missed abortion C. pseudocyesis D. threatened abortion E. twin intrauterine pregnancy

D. Fibroid uterus Patients with a fibroid uterus can have heavy, prolonged menses; bulk symptoms (pelvic pressure/pain); and an irregularly shaped, enlarged uterus. Treatment depends on symptom severity & desire for future fertility

40-year-old nulligravid woman presents due to heavy menstrual bleeding & pelvic pain. The patient has had 8-10 days of heavy bleeding with the passage of large clots every month. She has constant dull pelvic pressure unrelieved by ibuprofen. The patient had an appendectomy 4 years ago without complications. She stopped using OCP 3 years ago & is not sexually active. BMI is 23kg/m2. Physical exam reveals an enlarged, irregular shaped uterus & no other abnormalities. Which of the following is the most likely diagnosis? A. Adenomyosis B. Endometrial hyperplasia C. Endometriosis D. Fibroid uterus E. von Willebrand disease

B. Postpartum preeclampsia Postpartum preeclampsia, new-onset HTN with end-organ damage, can present up to 12 weeks postpartum & can be complicated by pulmonary edema. Treatment includes magnesium sulfate for seizure prophylaxis, antihypertensives & fluid restriction, oxygen support & diuretics for pulmonary edema.

41-year-old woman comes to the ED due to worsening SOB. The patient had an uncomplicated vaginal delivery 8 days ago & was discharged from the hospital 2 days after delivery. She says the dyspnea initially occurred only when walking, but now occurs at rest & when lying down. The patient has been breastfeeding her infant every 2-3 hours & is fatigued throughout the day. Temp is 37C (98.6F), BP is 166/98, HR is 70/min, RR are 24/min. Pulse ox is 92% on room air. Cardiac exam is normal. Auscultation of the lungs reveal diffuse bilateral rales. There is 3+ pitting edema of the knees. DTRs have multiple beats of clonus. Which of the following is the most likely diagnosis in this patient? A. Community-acquired pneumonia B. Postpartum preeclampsia C. Pulmonary embolism D. Pulmonary HTN E. Sheehan syndrome

E. Symptom diary PMS causes both physical & affective symptoms, which commonly include fatigue, bloating, hot flashes, mood swings & irritability. Diagnosis is supported with a symptom diary that reveals recurring symptoms 1-2 weeks prior to menses that resolve at cycle onset.

42-year-old woman comes to the office due to recurrent episodes of fatigue & hot flashes. The patient's most recent symptoms began 4 days ago; she says she feels "extremely cranky." She also has intermittent bloating & the symptoms sometimes cause her to miss work. Her LMP was 3 weeks ago. Menses occur every 30 days, with 3 days of heavy bleeding. The patient stopped taking OCPs last year after undergoing bilateral tubal ligation. She smokes a pack of cigarettes daily. Vital signs & PE are normal. Which of the following is the best next step in management for this patient? A. Benzo therapy B. FSH level C. Pelvic US D. Restart combined OCPs E. Symptom diary

D. Inflammatory breast carcinoma Inflammatory breast carcinoma is an aggressive form of breast cancer that typically presents with diffuse breast erythema, warmth, pain & edema with a peau d'orange appearance (dimpling).

42-year-old woman, gravida 3 para 3, comes to the office due to left breast swelling, pruritus & pain that has worsened over the past month. The patient weaned her youngest child from breastfeeding 2 months ago. She last saw a healthcare provider 1 month ago for mastitis & was prescribed ABX. Temp is 37.3C (99.1F). Breast exam shows a left breast that is diffusely warm, erythematous & edematous with some dimpling. The right breast appears normal. Which of the following is the most likely diagnosis? A. Breast abscess B. Fat necrosis C. Infiltrating ductal carcinoma D. Inflammatory breast carcinoma E. Intraductal papilloma

C. involuntary detrusor contractions urge incontinence is due to involuntary detrusor contractions. Treatment is with bladder training & pelvic floor exercises; antimuscarinics are recommended if symptoms persist.

43-year-old woman, gravid 3 para 3, comes to the office due to recurrent, strong desires to urinate. These symptoms began 4 months ago & now occur multiple times a day & once or twice a night. At times, the patient is unable to "hold in" her urine prior to sitting on the toilet, which interrupts her daily activities. She has no pain on urination. The patient has a new sexual parter. On exam, there is no leakage of urine when the patient is asked to cough. UA is normal. Which of the following is the most likely cause of this patient's incontinence? A. bladder outlet obstruction B. detrusor underactivity C. involuntary detrusor contractions D. urethral hypermobility E. viral urethritis

C. Pelvic US A palpable adnexal mass on PE is best evaluated by pelvic ultrasound to rule out malignant features. CA-125 testing has low specificity for initial screening of ovarian cancer in premenopausal patients.

46-year-ld nulliparous woman comes to the office for a routine exam. She has been feeling well. Her LMP was 1 week ago. The patient has regular menses every 30 days that lasts 3 days, with crampy abdominal pain on the 1st day relieved with Ibuprofen. The patient is sexually active with her husband; they do not use contraception as he has had a vasectomy. Abdominal exam shows a symmetric, nontender, non distended abdomen with normal bowel sounds. Speculum exam shows a cervix with a closed external os. Bimanual exam shows a 5-cm, irregular, right adnexal mass. Urine pregnancy test is negative. Which of the following is the best next step in management? A. CT scan of abdomen & pelvis B. MRI of abdomen & pelvis C. Pelvic US D. Reassurance & routine follow-up E. Serum CA-125 level

B. BMI endometrial adenocarcinoma can present with AUB & is diagnosed via endometrial Bx. Risk factors are related to excessive uterine estrogen exposure; the most common risk factors are obesity & chronic anovulation

47-year-old presents due to abnormal uterine bleeding (AUB). She has had several days of heavy bleeding every 2-3 weeks for the past 4 months. Menarche was at age 16 & patient was on estrogen/progestin-containing OCP during most of her 20s & 30s. She stopped taking OCPs at age 33 when she underwent a tubal ligation. She had a cervical confiscation at age 29 after an abnormal PAP smear, but all subsequent PAP tests have been normal. The patient drinks 2 cups of coffee in the morning & has no daily meds. BMI: 39kg/m2. An endometrial Bx shows endometrial adenocarcinoma. Which of the following is the most significant risk factor for this patient's Bx result? A. Age at menarche B. BMI C. Caffeine consumption D. OCP use E. Prior abnormal PAP smear

B. cervical cancer invasive cervical cancer can present with abnormal bleeding (postcoital spotting); a malodorous vaginal d/c; and a friable, exophytic cervical lesion

48-year-old woman comes to the office due to abnormal bleeding. The patient 1st noticed postcoital spotting 3 months ago. For the past month, the spotting has become increasingly heavy, requiring her to use a tampon or pad after intercourse & has been associated with a malodorous vaginal d/c. Pelvic exam shows a firm, exophytic, friable, 1-cm lesion that extends laterally on the posterior aspect of the cervix. A thin, watery, malodorous d/c is noted throughout the vaginal vault. Which of the following is the most likely diagnosis in this patient? A. acute cervicitis B. cervical cancer C. cervical polyp D. prolapsing leiomyoma E. trichomoniasis

C. Serum FSH level Menopause is a clinical diagnosis (cessation of menses for > 12 months) that usually does no require further eval in patients with previously normal menses & typically menopausal symptoms (dyspareunia due to vulvovaginal atrophy). Diagnosis in women without previously normal menses (prior hysterectomy) is confirmed by an elevated serum FSH level.

49-year-old woman comes to the office due to decreased interest in sex for the past 6 months. The patient has had a decreased libido for 6 months & now is having pain with intercourse (dyspareunia). Her only surgery was a hysterectomy at age 29 for severe endometriosis. Temp is 36.7 (98.1F), BP is 120/70, HR is 78/min. BMI is 23kg/m2. On pelvic exam, the vulva has scant pubic hair, & vulvovaginal mucosa is pale with clear vaginal d/c. The cervix & uterus are surgically absent & the ovaries are small. Which of the following is most likely to establish the diagnosis in this patient? A. Diagnostic laparoscopy B. PHQ-9 Depression screen C. Serum FSH level D. Serum testosterone level E. Vulvar biopsy

D. Selective serotonin reuptake inhibitor The most effective treatment for menopausal hot flashes is estrogen-based hormone replacement therapy. SSRIs & SNRIs are useful for patients who have contraindications to estrogen therapy (history of thromboembolism, breast cancer, coronary heart disease).

52-year-old woman comes to the office for eval of hot flashes. In recent months, they have been more intense and more frequent, occurring several times a day, and the patient has been waking up in the middle of the night bc her "sheets are drenched in sweat." Her LMP occurred at age 40, when she underwent a total abdominal hysterectomy without oophorectomy for uterine fibroids. The patient's medical history is also significant for an unprovoked deep venous thrombosis treated with anticoagulation at age 48. BMI is 24 kg/m2. Which of the following is the best next step in management of this patient? A. Low-dose clonidine B. Oral estrogen & progestin therapy C. Oral estrogen therapy D. Selective serotonin reuptake inhibitor E. Weight loss and vitamin E supplementation

C. Order a transvaginal pelvic US patients with postmenopausal bleeding require evaluation for endometrial cancer with either a transvaginal US (TVUS) or endometrial biopsy. In women who initially undergo a TVUS, those with an endometrium < 4mm require no additional evaluation. In contrast, women with an endometrium of > 4 mm require an endometrial biopsy

54-year-old woman presents due to several months of irregular vaginal spotting. Menopause occurred over a year ago & was complicated by severe hot flashes & night sweats. The patient was prescribed a transdermal estrogen patch & cyclic progesterone-only pills. She has been using the estrogen patches consistently but stopped taking the pills due to severe nausea. A PAP 2 years ago was normal. VS are normal. BMI: 23kg/m2. On speculum exam, there are no cervical or vaginal lesions & there is no blood in the vagina. Bimanual exam reveals a small, mobile uterus & no adnexal masses. Which of the following is the best next step in management of this patient? A. Gynecology referral for a hysterectomy with bilateral saplings-oophorectomy B. Gynecology referral for an endometrial ablation C. Order a transvaginal pelvic US D. Place a progesterone-containing IUD E. Switch from transdermal patch to vaginal estrogen

E. Paget Disease Paget disease of the breast is characterized by a unilateral, erythematous, intensely pruritus, ulcerative lesion of the nipple-areolar complex. Diagnosis is by tissue biopsy. Patients also require diagnostic bilateral mammography, as many have underlying breast malignancy.

56-year-old woman comes to the office due to breast pruritus. The patient reports increasing pruritus around the right areola over the last 3 months that has not resolved with topical emollients. The pruritus is continuous & she is constantly scraping the breast. The patient has now noticed a lesion around the nipple. She has had no change in skin products or detergents. The patient has DM2. Her last mammogram, 2 years ago, was normal. The right breast has an ulcerative lesion over the right nipple and areola that does not extend to the surrounding tissue; there are no palpable masses or axillary lymph nodes. The left breast is normal. Which of the following is the most likely diagnosis in this patient? A. Candida mastitis B. Contact dermatitis C. Ductal extasia D. Intraductal papilloma E. Paget disease

C. Core needle biopsy women age > 30 with a palpable breast mass & negative breast imaging require further examination with a core needle biopsy to exclude malignancy

57-year-old postmenopausal woman present for her AWW. She is feeling well with no concerns. Her maternal aunt died from metastatic breast cancer at age 72. VS are normal. BMI: 29kg/m2. Breast exam reveals a nontender, fixed 3-cm mass in the upper outer quadrant of the left breast. There are no skin changes, nipple discharge or associated axillary lymphadenopathy. A diagnostic mammogram & breast US Reveal no masses or calcification. Which of the following is the best next step in management of this patient? A. BRCA testing B. Clinical breast exam & repeat imaging in 1 year C. Core needle biopsy D. MRI of breast E. Tamoxifen

C. overflow incontinence Overflow incontinence is caused by underachieve detrusor function or bladder outlet obstruction. Anticholinergic (tricyclic antidepressant, antihistamine, antimuscarinics) relax the detrusor muscle, increase bladder capacity & cause urinary retention. Chronic urinary retention leads to dribbling, incomplete emptying & a large postpaid residual volume.

58-year-old woman comes to the office due to leakage of urine. The patient has had increased urinary frequency over the last 3 months, often void every hour and multiple times each night. She has difficulty initiating urination & voids a small volume of urine. The patient now has continued leakage of urine. She has depression and switched from sertraline to amitriptyline 4 months ago. Abdominal exam reveals a nontender, suprapubic mass. Pelvic exam reveals continuous leakage of urine from the urethra; no pooling is seen in the vaginal vault. A postpaid residual volume is elevated. UA is normal. Which of the following is the most likely diagnosi? A. Bladder cancer B. interstitial cystitis C. overflow incontinence D. urgency incontinence E. vesicovaginal fistula

D. Needle aspiration Breast abscesses present with a unilateral fluctuant, tender, palpable breast mass with fever, surrounding erythema/pain, malaise, & associated axillary LAD. Management includes drainage (needle aspiration) & empiric ABX (Dicloxacillin, Cephalexin).

6 weeks after a spontaneous, uncomplicated term vaginal delivery, a 32-year-old woman, gravida 1 para 1 comes to the ED due to right breast pain. The patient 1st noticed pain & redness on her right breast 1 week ago. She has continued to breast feed her infant from the unaffected breast. The patient has no chronic medical conditions & her only med is a multivitamin. Temp is 38.3 (101F). PE shows an area of erythema extending from the areola to the lateral edge of the right breast & surrounding a well-circumscribed, 4-cm area of fluctuance. Axillary LAD is present. Which of the following is the best next step in management of this patient? A. Breast binding & Acetaminophen B. Core needle & skin punch biopsies C. Ice packs & use of a supportive bra D. Needle aspiration & ABX E. Warm compress & massage

B. Estrogen deficiency GU syndrome of menopause (or atrophic vaginitis) causes an estrogen deficiency that results in urogenital atrophy & can present with multiple urinary symptoms (dysuria, urgency incontinence, recurrent UTIs). Treatment is with non hormonal moisturizers; vaginal estrogen is prescribed for non improving or severe symptoms.

60-year-old woman comes to the clinic due to the persistent feeling of "always having to go to the bathroom." For the past 3 years, the patient has had urinary urgency followed by involuntary loss of urine 2-3 times per day. She has dysuria & nocturne, but she does not lose urine while coughing or sneezing. Menopause occurred at age 52. She has had recurrent UTIs for the past year. On PE, the vulvar skin shows reduced elasticity with labia minor retraction. The vaginal epithelium is smooth, dry & has areas of patchy erythema. On Valsalva maneuver, there is no loss of urine. UA & postvoid residual urine testing is normal. Which of the following is the most likely cause of this patient's urinary symptoms? A. Chronic bladder pain syndrome B. Estrogen deficiency C. Incomplete bladder emptying D. Urethral hypermobility E. Vesicovaginal fistula formation

E. Vaginal Estrogen Vulvovaginal Atrophy (GU syndrome of menopause) typically presents with dyspareunia due to vaginal Introits narrowing. Treatment is with water-based lubricants & vaginal estrogen.

63-year-old postmenopausal woman comes to the office due to pain during sex (dyspareunia). The patient had been abstinent since the death of her husband 10 years ago & attempted intercourse recently with a new partner. Although she uses lubrication, the patient has severe pain with intercourse. She also has some post-coital spotting & mild vaginal pruritus. The patient occasionally has small amounts of thin, malodorous vaginal d/c. Exam of the external genitalia shows scant pubic hair & pale, thin skin. The Introits is narrow & the vaginal mucosa is pale, smooth, dry & slightly friable. Vaginal pH is 6. Wet mount microscopy shows a few inflammatory cells, no clue cells & no trichomonads. Which of the following is the best next step in management of this patient? A. Clobetasol B. Fluconazole C. Hormone replacement therapy D. Metronidazole E. Vaginal Estrogen

C. Reassurance & observation Pelvic organ prolapse may present with pelvic pressure or urinary dysfunction (retention, stress urinary incontinence). However, many patients with this are asymptomatic & are managed with reassurance & observation only.

63-year-old woman comes to the office for a routine annual exam. The patient feels well & has no concerns. She underwent menopause at age 50. The patient exercises multiple days a week & has no urinary or fecal leakage. On pelvic exam, vulvar atrophy is present & the vagina appears pale but has no lesions. The cervix appears normal. On Valsalva manuever, there is a bulge of the anterior vaginal wall to the Introitus. Postvoid bladder & renal US is normal. Which of the following is the best next step in management of the patient's pelvic organ prolapse? A. Hormone replacement therapy B. Pessary placement C. Reassurance & observation D. Surgical repair E. Urodynamic testing

B. No further screening Pap testing may be discontinued in patients age 65 who have had adequate Pap testing without prior high-grade cervical lesions or cervical cancer risk factors (tobacco use, immunocompromised status).

67-year-old woman comes to the office for a routine health exam. The patient feels well. Her LMP was at age 53 with no bleeding since. Pap test & HPV cotesting were negative 2 years ago at age 65. All prior Pap tests were normal. The patient is sexually active & in a monogamous relationship with her husband of 42 years. She has no medical conditions & does not use tobacco, alcohol or illicit drugs. Pelvic exam shows a normal cervix without lesions; a smooth, anteverted, mobile uterus; and no adnexal masses. Which of the following is the most appropriate recommendation for cervical cancer screening in this patient? A. HPV testing only at this visit B. No further screening C. Pap & HPV testing at this visit D. Pap test only at this visit E. Repeat Pap test in 3 years

D. Pelvic organ prolapse Severe or prolonged pelvic organ prolapse can cause vaginal erosions that lead to abnormal vaginal bleeding (postmenopausal bleeding, postcoital bleeding). Treatment of prolapse is via pessary or surgical correction; erosions can be treated with vaginal estrogen.

68-year-old woman comes to the office due to vaginal spotting. The patient has had dark brown blood & occasionally has bright red blood on her pad. For the past year she has been sexually active with a new partner & recently noticed some postcoital bleeding. The patient has no dyspareunia, abnormal vaginal d/c, dysuria or hematuria. Vital signs are normal. BMI is 30kg/m2. Speculum exam shows an erosion near the posterior fornix. The cervix is erythematous but there are no lesions. The uterus is small, mobile & nontender. There are no adnexal masses or tenderness. On Valsalva, a mass protrudes past the hymeneal ring. Which of the following is the most likely diagnosis for this patient? A. Cervical cancer B. Endometrial polyp C. Endometritis D. Pelvic organ prolapse E. Uterine leiomyoma

E. vulvar biopsy Vulvar lichen sclerosis is a risk factor for the development of vulvar cancer. Therefore, patients with a history of lichen sclerosis & a new univocal vulvar lesion (firm, white plaque) require vulvar biopsy to evaluate for vulvar cancer

70-year-old woman presents with c/o vulvar pruritus. She has had increasing pruritus over the past 4 months that has not improved with topical emollients. She has no pelvic pain, vaginal bleeding or abnormal d/c. She had similar symptoms 20 years ago, for which she was prescribed a CCS cream that resolved the symptoms. BMI: 32kg/m2. On pelvic exam, there is a firm, white plaque with overlying excoriations on the left labium magus. Mild atrophy of the bilateral labia minor is noted & the vagina has minimal rogation but no lesions. Which of the following is the best next step in management of this patient? A. high-dose topical CCS B. oral anti fungal med C. topical estrogen cream D. trichloroacetic acid therapy E. vulvar biopsy

C. 45-year-old who has had normal Pap test without HPV tests, the most recent of which was 3 years ago Current guidelines recommend initiating routine cervical cancer screening at age 21. Recommended screening strategies for women age 30-65 include: Pap test plus HPV testing every 5 years; primary HPV testing every 5 years; Pap tests with reflex HPV testing every 3 years; or Pap only every 3 years. Screening can be d/c in most women who are age 65 or older of have had a total hysterectomy

Which of the following women requires cervical cancer screening as part of a health maintenance exam? A. 19-year-old who has never had a Pap test or HPV test & has had > 3 sex partners during the past 2 years B. 38-year-old who has had a normal Pap test & negative HPV tests, the most recent of which was 4 years ago C. 45-year-old who has had normal Pap tests without HPV tests, the most recent of which was 3 years ago D. 55-year-old who has had normal Pap tests, the most recent of which was prior to a total hysterectomy for a uterine leiomyoma 9 years ago E. 72-year-old who has had normal Pap tests, the most recent of which was 7 years ago


Ensembles d'études connexes

AD Banker Ch.15 - Ethics and Law

View Set

2023 State Insurance Statutes, Rules, and Regulations

View Set

American Government: Reading Quiz 2

View Set